Respiratory Flashcards

1
Q

Define Chronic Bronchitis

A

Bronchitis means inflammation of the bronchial tubes in the lung.

It is said to be chronic when it causes a productive cough for at least 3 months each year for 2 or more years.

How well did you know this?
1
Not at all
2
3
4
5
Perfectly
2
Q

Epidemiology of Bronchitis

A

Usually co-exists with emphysema, causing COPD.

How well did you know this?
1
Not at all
2
3
4
5
Perfectly
3
Q

RF for Bronchitis

A
  • Smoking
  • Exposure to air pollutants e.g. sulfur and nitrogen dioxide
  • Exposure to dust and silica
  • Family history of chronic bronchitis
How well did you know this?
1
Not at all
2
3
4
5
Perfectly
4
Q

Pathophysiology of Bronchitis

A

The airways are exposed to all sorts of irritants and chemicals.

This can result in infiltration of the walls with inflammatory cells.

The epithelial layer may become ulcerated and, with time, squamous epithelium replaces the columnar cells (squamous metaplasia) when the ulcer heals.

The inflammation is followed by scarring and thickening of the walls, which narrows the small airways.

Also, the irritants stimulate hypertrophy and hyperplasia of the mucinous glands in the main bronchi, as well as the goblet cells in the bronchioles, which increases mucus production in both locations.

Since the bronchioles are smaller, even a slight increase in mucus can lead to airway obstruction, which contributes to the majority of the air trapping.

The reid index is a measurement (usually done post-mortem) to show the ratio of the thickness of the bronchial mucinous glands, relative to the total thickness of the airway - from the epithelium to the cartilage. Normally, this ratio should be less than 40%, but it can be over 40% for people with chronic bronchitis, due to hyperplasia and hypertrophy of the glands.

Also, smoking makes the cilia short and less mobile, making it harder to move mucus up and out of bronchioles. A cough is sometimes the only way to clear this mucus.

People with chronic bronchitis also often present with hypoxemia and hypercapnia. This is because the mucus plugs block airflow, causing high levels of CO2 and low levels of O2 in the lung so less O2 moves into blood and less CO2 moves out of blood.

Blood vessels can then undergo vasoconstriction to shunt blood away from damaged tissue towards healthy lung tissue.

Overall, there is airway narrowing due to hyperplasia, inflammation and oedema.

How well did you know this?
1
Not at all
2
3
4
5
Perfectly
5
Q

Effects on FVC, FEV1, FEV1:FVC and TLC of COPD

A

In COPD, the airways become obstructed and the lungs don’t empty properly which leaves air trapped inside the lungs.

  • FVC (max air exhaled in one breath): lowered
  • FEV1 (first second of air breathed out in a single breath): lowered, more than the FVC
  • FEV1:FVC ratio: lowered
  • TLC (total lung capacity): increased due to air trapping
How well did you know this?
1
Not at all
2
3
4
5
Perfectly
6
Q

Signs of Bronchitis

A
  • Wheeze: due to narrowing of the passageway available for air to move in and out
  • Crackles or rales: caused by the popping open of small airways
  • Cyanosis (blue bloaters): if there is buildup of CO2 in blood
How well did you know this?
1
Not at all
2
3
4
5
Perfectly
7
Q

Symptoms of Bronchitis

A
  • Productive cough
  • Dyspnoea
  • Signs of CO2retention
    • Drowsy
    • Asterixis
    • Confusion
How well did you know this?
1
Not at all
2
3
4
5
Perfectly
8
Q

Management of Bronchitis

A

Examples include:

  • Smoking cessation
  • Management of associated illnesses
  • Antibiotics for infections
  • Supplemental oxygen
  • Bronchodilators
  • Inhaled steroids
How well did you know this?
1
Not at all
2
3
4
5
Perfectly
9
Q

Complications of Bronchitis

A
  • Cor pulmonale and right sided heart failure: vasoconstriction leads to pulmonary hypertension which affects the functioning of the right ventricle.
  • Lung infections: can develop behind the mucus plugs
How well did you know this?
1
Not at all
2
3
4
5
Perfectly
10
Q

Define Emphysema

A

In emphysema, the alveolar air sacs become damaged or destroyed.

The alveoli permanently enlarge and lose elasticity, and as a result, individuals typically have difficulty with exhaling, which depends heavily on the ability of lungs to recoil.

How well did you know this?
1
Not at all
2
3
4
5
Perfectly
11
Q

Epidemiology of Emphysema

A

Usually co-exists with chronic bronchitis, causing COPD

How well did you know this?
1
Not at all
2
3
4
5
Perfectly
12
Q

RF for Emphysema

A

Smoking

How well did you know this?
1
Not at all
2
3
4
5
Perfectly
13
Q

Pathophysiology of Emphysema

A

Normally, oxygen flows out of the alveoli and into the blood while carbon dioxide makes the reverse commute.

When the lung tissue is exposed to irritants e.g. cigarette smoke, it triggers an inflammatory reaction.

Inflammatory reactions attract various immune cells which release inflammatory chemicals as well as proteases e.g. elastases and collagenases.

Normally, there is a low pressure environment in the airways and elastin in the walls prevents the lungs from collapsing inwards in this low pressure environment. This allows air to be exhaled out.

However, in emphysema, the elastin is lost which causes collapse. This results in:

  • Air-trapping distal to the point of collapse
  • The lungs becoming more compliant - when air is inhaled, the lungs easily expand and hold onto that air.
  • Breakdown of the thin alveolar walls - septa. This reduces the surface area for gas exchange.

Over time, as more and more lung tissue is affected, emphysema can lead to hypoxemia.

This leads to hypoxic vasoconstriction to navigate blood flow away from damaged lung tissue.

How well did you know this?
1
Not at all
2
3
4
5
Perfectly
14
Q

Types of Emphysema

A

Types of emphysema:

Different types affect the acinus (endings of the lung containing alveoli) in different ways:

  • Centriacinar emphysema: most common pattern. Only damages the central or proximal alveoli of the acinus. It also typically affects the upper lobes of the lungs. This is usually due to smoking.
  • Panacinar emphysema: entire acinus is uniformly affected and typically affects the lower lobes of the lungs. Often associated with alpha-1 antitrypsin deficiency. Alpha-1 antitrypsin is a protease inhibitor and protects collagen and elastin. Without it, the proteases are able to break down the alveolar walls
  • Paraseptal emphysema: distal alveoli of the acinus are most affected. Typically affects the lung tissue on the periphery of the lobules.
  • Irregular emphysema: scarring and damage that affects the lung parenchyma
    patchily, independent of acinar structure
How well did you know this?
1
Not at all
2
3
4
5
Perfectly
15
Q

Signs of Emphysema

A
  • Breathing with pursed lips (pink puffers): prevents alveolar collapse by increasing the positive end expiratory pressure
  • Barrel shaped chest: due to air trapping and hyperinflation
  • Loss of cardiac dullness:due to hyperexpansion of lungs from emphysema
  • Downward displacement of liver:due to hyperexpansion of lungs from emphysema
  • On imaging: increased anterior-posterior diameter, a flattened diaphragm, and increased lung field lucency
How well did you know this?
1
Not at all
2
3
4
5
Perfectly
16
Q

Symptoms of Emphysema

A
  • Dyspnoea
  • Cough: could be productive
  • Weight loss: due to energy expenditure while breathing
  • Signs of CO2retention
    • Drowsy
    • Asterixis
    • Confusion
How well did you know this?
1
Not at all
2
3
4
5
Perfectly
17
Q

Management of Emphysema

A

Examples include:

  • Smoking cessation
  • Supplemental oxygen
  • Bronchodilators
  • Inhaled steroids
  • Antibiotics: for secondary infections
How well did you know this?
1
Not at all
2
3
4
5
Perfectly
18
Q

Complications of Emphysema

A
  • Pneumothorax: the distal enlarged alveoli in paraseptal emphysema can rupture and cause a pneumothorax.
  • Cor pulmonale and right-sided heart failure: extensive vasoconstriction causes pulmonary hypertension and puts pressure on right ventricle
How well did you know this?
1
Not at all
2
3
4
5
Perfectly
19
Q

Define COPD

A

Chronic obstructive pulmonary disease (COPD) describes progressive and irreversible obstructive airway disease.

It is a combination of emphysema and chronic bronchitis.

How well did you know this?
1
Not at all
2
3
4
5
Perfectly
20
Q

Epidemiology of COPD

A
  • There are 1.2 million people with COPD living in the UK
  • COPD is the fourth leading cause of death globally
  • Usually diagnosed >45 years
How well did you know this?
1
Not at all
2
3
4
5
Perfectly
21
Q

RF for COPD

A
  • Age:usually diagnosed after the age of 45
  • Tobacco smoking: the single greatest risk factor for COPD
  • Air pollution
  • Occupational exposure: such as dust, cadmium (in smelting), coal, cotton, cement and grain
  • Alpha-1 antitrypsin deficieny: younger patients present with features of COPD
How well did you know this?
1
Not at all
2
3
4
5
Perfectly
22
Q

Pathophysiology of COPD

A

COPD is a combination ofemphysema and chronic bronchitis

  • Emphysemainvolves loss of alveolar integrity due to an imbalance between proteases and protease inhibitors (e.g. alpha-1 antitrypsin) triggered by chronic inflammation, such as smoking, which causes elastin breakdown
  • Bronchitisinvolves increased mucus secretion secondary to ciliary dysfunction and increased goblet number and size → lung parenchymal destruction → impaired gas exchange

In COPD, the airways become obstructed and the lungs don’t empty properly which leaves air trapped inside the lungs.

  • FVC (max air exhaled in one breath): lowered
  • FEV1 (first second of air breathed out in a single breath): lowered, more than the FVC
  • FEV1:FVC ratio: lowered
  • TLC (total lung capacity): increased due to air trapping
How well did you know this?
1
Not at all
2
3
4
5
Perfectly
23
Q

V/Q mismatch in regards to COPD

A

V/Q (ventilation perfusion) mismatch is partly due to damage and mucus plugging of smaller airways from the chronic inflammation and partly due to rapid closure of smaller airways in expiration owing to the loss of elastic support - this mismatch leads to a fall in PaO2 and increased respiration. Usually PaCO2 is unaffected until patient’s are unable to maintain their respiratory efforts. Buildup of CO2 leads to cyanosis.

At first, excess CO2 is the drive for respiration. As patient’s become desensitised to CO2, hypoxaemia becomes the drive for respiration.

How well did you know this?
1
Not at all
2
3
4
5
Perfectly
24
Q

Exacerbations of COPD

A

Exacerbations: patients are susceptible to exacerbations during which there is worsening of their lung function. Exacerbations are often triggered by infections and these are called infective exacerbations. (Refer to ‘other notes’ below)

How well did you know this?
1
Not at all
2
3
4
5
Perfectly
25
Q

Signs of COPD

A
  • Tachypnoea
  • Barrel chest
  • Hyperresonance on percussion
  • Quiet breath sounds and wheeze
  • Pursing of lips during expiration: helps to prolong expiration to breathe out as much air as possible.
  • Cyanosis
  • Tar staining of fingers
  • Loss of cardiac dullness:due to hyperexpansion of lungs from emphysema
  • Downward displacement of liver:due to hyperexpansion of lungs from emphysema
  • Evidence of an exacerbation:
    • Significant dyspnoea, wheeze and cough
    • Coarse crepitations
    • Pyrexia
  • Evidence of cor pulmonale: e.g. peripheral oedema
How well did you know this?
1
Not at all
2
3
4
5
Perfectly
26
Q

Symptoms of COPD

A
  • Dyspnoea: particularly on exertionMRC dyspnoea scale is used to grade the severity of breathlessness:
    1. Breathlessness on strenuous exercise.
    2. Breathlessness on hurrying or slight hill.
    3. Walksslower than contemporarieson ground level due to breathlessness ORhave to stop to catch breath when walking at own pace.
    4. Stops to catch breath after 100 metres ORa few minutes of walking
    5. Breathlessness on minimal activity (dressing) or unable to leave the house due to breathlessness
  • Productive cough
  • Wheeze
  • Chest tightness
  • Weight loss: due to energy expenditure while breathing
  • Signs of CO2retention
    • Drowsy
    • Asterixis
    • Confusion
How well did you know this?
1
Not at all
2
3
4
5
Perfectly
27
Q

Primary investigations for COPD

A
  • Diagnosis is usually based on clinical presentation:A diagnosis of COPD should be considered in any patient over 35 years old who are current or ex-smokers with one or more symptoms of COPD e.g. exertional breathlessness, recurrent sputum production, wheeze, ‘winter bronchitis’ or chronic cough
  • Primary investigations
    • Spirometry and bronchodilator reversibility (BDR): FEV1/FVC<0.70; bronchodilator will not reverse symptoms.
    • Chest X-ray: flattened diaphragm, hyperinflation and bullae. Should also be performed to see if there is evidence of lung cancer.
    • FBC: COPD causes chronic hypoxia which may result in secondary polycythaemia; also required to determine if eosinophilia is present.
    • Calculate body mass index (BMI): as a baseline to later assess weight loss (e.g. cancer or severe COPD) or weight gain (e.g. steroids)
How well did you know this?
1
Not at all
2
3
4
5
Perfectly
28
Q

Management of COPD

A
  • General:
    • Smoking cessation: offer nicotine replacement, varenicline or bupropion
    • Pulmonary rehabilitation: for patients who are self-perceived as functionally disabled by COPD (e.g. MRC grade ≥3)
    • Vaccinations: one-offpneumococcaland annualinfluenza
    • Good diet and exercise: esp if obese
  • Key pharmacological management:
    • Bronchodilators
      • SABA:short-acting beta-agonist (e.g. salbutamol)
      • SAMA: short-acting muscarinic antagonist (ipratropium)
      • LABA: long-acting beta-agonist (e.g. salmeterol)
      • LAMA: long-acting muscarinic antagonist (e.g. tiotropium)
    • ICS: inhaled corticosteroid (e.g. beclometasone)
How well did you know this?
1
Not at all
2
3
4
5
Perfectly
29
Q

Complications of COPD

A
  • Pulmonary hypertension and cor pulmonale: chronic hypoxia leads to pulmonary vasoconstriction and hypertension. The right side of the heart has to pump against high pressures and eventually fails.Clinical features include peripheral oedema, raised JVP, hepatomegaly, parasternal heave and a loud P2
  • Pneumothorax:common due to bullae formation, resulting in a secondary spontaneous pneumothorax
  • Respiratory failure: type 1 or type 2
    • Normal pCO2withlow pO2indicatestype 1 respiratory failure(onlyoneis affected)
    • Raised pCO2withlow pO2indicatestype 2 respiratory failure(twoare affected)
  • Exacerbation:usually due to bacterial or viral infection
  • Secondary polycythaemia: chronic hypoxia induces polycythaemia
  • Infections: people with COPD are more prone to infections
How well did you know this?
1
Not at all
2
3
4
5
Perfectly
30
Q

Prognosis for COPD

A

In 2012, 5.3% of all UK deaths were due to COPD, with over 90% of COPD-related deaths occurring in the over-65s age group.

Smoking cessation is the most important intervention in improving survival in stable COPD.

Other factors that improve survival in stable COPD include long term oxygen therapy (if appropriate) and lung volume reduction surgery (if appropriate).

How well did you know this?
1
Not at all
2
3
4
5
Perfectly
31
Q

Investigations for exacerbation of COPD

A
  • ABG: shows respiratory acidosis (low PH and increased retention of CO2); increased bicarbonate suggests compensation by the kidney
  • Chest xray:to look for pneumonia or other pathology
  • ECG:to look for arrhythmia or evidence of heart strain (heart failure)
  • FBC:to look for infection (raised white cells)
  • U&E:to check electrolytes which can be affected by infection and medications
  • Sputum culture:if significant infection is present
  • Blood cultures:if septic
How well did you know this?
1
Not at all
2
3
4
5
Perfectly
32
Q

Management of exacerbation of COPD

A

Requires additional therapy above the baseline medications

If well enough to remain at home:

  • Prednisolone30mg once daily for 7-14 days
  • Regularinhalersor homenebulisers
  • Antibioticsif there is evidence of infection

In hospital:

  • Nebulised bronchodilators(e.g. salbutamol 5mg/4h and ipratropium 500mcg/6h)
  • Steroids(e.g. 200mg hydrocortisone or 30-40mg oral prednisolone)
  • Antibioticsif evidence of infection
  • Physiotherapycan help clear sputum

Options in severe cases not responding to first line treatment:

  • IVaminophylline
  • Non-invasive ventilation(NIV)
  • Intubationandventilationwith admission to intensive care
  • Doxapramcan be used as a respiratory stimulant where NIV or intubation is not appropriate
How well did you know this?
1
Not at all
2
3
4
5
Perfectly
33
Q

Define Asthma

A

Asthma is a chronic inflammatory airway disease characterised by intermittent airway obstruction and hyper-reactivity.

There are 2 types:

  • Allergic/ eosinophilic: allergens and atopy
  • Non-allergic/ non-eosinophilic: e.g. exercise, cold air and stress
How well did you know this?
1
Not at all
2
3
4
5
Perfectly
34
Q

Epidemiology of Asthma

A
  • Asthma is a common disease with a prevalence of almost 10% in the US
  • Commonly starts in childhood between the ages 3-5 years and may either worsen or improve during adolescence
  • Peak prevalence between 5-15 years
How well did you know this?
1
Not at all
2
3
4
5
Perfectly
35
Q

RF for Asthma

A
  • History of atopy: such as eczemaand allergic rhinitis (IgE-mediated atopic conditions)
  • Family history
  • Allergens: such as tobacco smoke, pets, outdoor air pollution, weeds, grass, pollen and dust mites
  • Viral upper respiratory tract infection
  • Other triggers: cold weather and exercise, medications e.g. beta blockers and aspirin
  • Occupational exposure (10-15%): isocyanates are the most common cause ofoccupational asthma(e.g. spray painting). Other causes include flour (bakers), platinum salts, soldering flux resin, glutaraldehyde and epoxy resins. These particularly affect people involved in plastic, foam and glue manufacturing. Requires a specialist referral.
How well did you know this?
1
Not at all
2
3
4
5
Perfectly
36
Q

Pathophysiology/Aetiology of Asthma

A

In asthma there is often an excessive reaction from Th2 cells against specific allergens.

Allergens from environmental triggers e.g. cigarette smoke, are picked up by dendritic cells and presented to Th2 cell. This leads to production of cytokines e.g. (IL-3, IL-4, IL-5, IL-10, IL-13)

This leads to the production of IgE antibodies which coat mast cells and stimulate them to release granules containing things e.g. histamines, leukotrienes and prostaglandins.

It also results in the activation of eosinophils which promote an immune response by releasing more cytokines and leukotrienes.

Minutes after the exposure to the allergen, smooth muscle around the bronchioles start to spasm and there is increased mucus secretion. This narrows the airways making it difficult to breathe.

There is also an increase in vascular permeability and recruitment of additional immune cells from the blood. A few hours after exposure, these immune cells, release chemical mediators that physically damage the endothelium of the lungs.

Initially these inflammatory changes are completely reversible, but over the years irreversible changes start to take place, leading to thickening of the epithelial basement membrane, which permanently reduces the airway diameter.

Asthma is thought to involve a complex interaction between genetic and environmental factors:

  • Genetic susceptibilitypredisposes patients toairway hyper-responsiveness, triggered by environmental factors such as viral infection, allergens (the main cause in children), cold and exercise
    • Genes controlling the production of cytokines IL-3,-4,-5,-9 & -13
    • ADAM33 is associated with airway hyper-responsiveness and tissue remodelling
  • In general, causes of childhood asthma diagnosed before age 12 are thought to be due to a stronger genetic influence, whereas later onset asthma is more likely to be largely due to environmental factors.
  • The hygiene hypothesis: reduced early immune-system exposure to bacteria and viruses might increase the risk of later developing asthma, possibly by altering the overall proportion of immune cell subtypes.
How well did you know this?
1
Not at all
2
3
4
5
Perfectly
37
Q

Signs of Asthma

A
  • Diurnal PEFR variation: worse at night and early morning
  • Dyspnoea and expiratory wheeze
  • Samter’s triad
    • Nasal polyps
    • Aspirin insensitivity
    • Asthma
How well did you know this?
1
Not at all
2
3
4
5
Perfectly
38
Q

Symptoms of Asthma

A
  • Episodic shortness of breath: diurnal variation (worse at night and early morning)
  • Dry cough
  • Wheeze and ‘chest tightness’
  • May be sputum
  • History of exposure to a trigger
How well did you know this?
1
Not at all
2
3
4
5
Perfectly
39
Q

Primary investigations of Asthma

A
  • Fractional exhaled nitric oxide (FeNO):>40 ppb is positive in adults
  • Spirometry:FEV1/FVC <70% suggests obstruction. If obstruction is found, BDR should be carried out
    • Bronchodilator reversibility (BDR): improvement of FEV1 by ≥12%andincrease ≥200ml in volume post-bronchodilator
How well did you know this?
1
Not at all
2
3
4
5
Perfectly
40
Q

Stepwise management for asthma

A

Step 1: Newly diagnosed asthma -
SABA

Step 2: Not controlled on previous step OR newly diagnosed asthma with symptoms >3/week or night time waking -
SABA + low dose ICS (e.g. Budesonide)

Step 3:
SABA + low dose ICS + LTRA (e.g. montelukast)

Step 4:
SABA + low dose ICS + LABA (e.g. salmeterol)

If it gets worse add MART or LAMA or theophylinine

How well did you know this?
1
Not at all
2
3
4
5
Perfectly
41
Q

Complications of asthma

A
  • Asthma exacerbations:typically triggered by an upper respiratory tract infection, pneumonia, or exposure to a trigger, e.g. an allergen or occupational exposure
  • Pneumothorax
  • Oral thrush: due to steroid medication
How well did you know this?
1
Not at all
2
3
4
5
Perfectly
42
Q

Prognosis for asthma

A

The life expectancy of controlled asthma is similar to the general population.

Remission rates are low and usually seen in mild cases.

How well did you know this?
1
Not at all
2
3
4
5
Perfectly
43
Q

Define Asthma exacerbation

A

An asthma exacerbation is an acute or subacute episode of progressive worsening of symptoms of asthma, including shortness of breath, wheezing, cough, and chest tightness.

How well did you know this?
1
Not at all
2
3
4
5
Perfectly
44
Q

Pathophysiology of asthma exacerbation

A

Asthma is a chronic, reversible obstructive airway disease. The onset is typically in childhood but often persists into adulthood.

Patients with asthma have hyperreactive airways which, in response to certain triggers, can undergo bronchoconstriction and inflammation, with excessive mucous secretion.

The inflammatory response is complex but is usually driven by Th2 cells, particularly if the trigger is an allergen.

The result of this is an asthma exacerbation, also known as acute asthma.

Key triggers:

  • Known diagnosis of asthma
  • Allergen exposure:pollen, dust mite, pets
  • Occupationalexposures:plastics, foam, glue, flour
  • Viral infection
  • Smokingexposure
  • Pollution
  • Exercise
How well did you know this?
1
Not at all
2
3
4
5
Perfectly
45
Q

General clinical manifestations of Asthma exacerbation

A
  • Progressively worsening shortness of breath
  • Use of accessory muscles
  • Fast respiratory rate (tachypnoea)
  • Symmetrical expiratory wheeze on auscultation
  • The chest can sound “tight” on auscultation with reduced air entry
How well did you know this?
1
Not at all
2
3
4
5
Perfectly
46
Q

Signs of Moderate asthma exacerbation

A

PEFR 50-75%

How well did you know this?
1
Not at all
2
3
4
5
Perfectly
47
Q

Signs of Severe Asthma exacerbation

A

Any of the following:

  • PEFR 33-50%
  • Respiratory rate ≥ 25
  • Heart rate ≥ 110
  • Inability to complete sentences in one breath
How well did you know this?
1
Not at all
2
3
4
5
Perfectly
48
Q

Signs of Life-threatening asthma exacerbation

A

Any of the following in a patient with severe asthma:

  • PEFR < 33%
  • SpO2< 92%
  • PO2< 8 kPa
  • PCO2normal (4.0-6.0 kPa)
  • Altered consciousness
  • Exhaustion
  • Arrhythmia
  • Hypotension
  • Cyanosis
  • Silent chest: no wheeze. This occurs when the airways are so tight that there is no air entry at all.
  • Poor respiratory effort
How well did you know this?
1
Not at all
2
3
4
5
Perfectly
49
Q

Signs of near fatal asthma exacerbation

A

High pCO2 (> 6.0 kPa) and/or requiring ventilation with raised inflation pressures.

How well did you know this?
1
Not at all
2
3
4
5
Perfectly
50
Q

Investigations for asthma exacerbation

A
  • Peak flow expiratory rate (PEFR)
    • Moderate:> 50-75% of baseline or predicted
    • Severe:33-50% of baseline or predicted
    • Life-threatening:< 33% of baseline or predicted
  • ABG:patients will initially have respiratory alkalosis. Abnormal or high PCO2is an extremely concerning sign as it implies patient is tiring.
  • Inflammatory markers:raised CRP and WCC may suggest an infective trigger
  • CXR:hyperexpansion +/- a focus of infection
How well did you know this?
1
Not at all
2
3
4
5
Perfectly
51
Q

Management of asthma exacerbation in a patient with moderate asthma

A
  • Moderate asthma: treated in primary care with
    • Inhaled salbutamol
    • 5-day course of prednisolone
How well did you know this?
1
Not at all
2
3
4
5
Perfectly
52
Q

Immediate management for severe or life threatening asthma exacerbation

A
  • Oxygen:aim for SpO294-98%
  • Nebulised bronchodilators:
    • Salbutamol 5mg is first-line and if the patient fails to improve, it can be given continuously.
    • Ipratropium bromide 0.5mg can be given additionally every 4-6 hours
  • Corticosteroids:
    • Prednisolone 40mg OD or
    • 100mg hydrocortisone IV if the oral route is not tolerated (e.g. vomiting or reduced consciousness)
    • Patients should continue on their inhaled corticosteroid
How well did you know this?
1
Not at all
2
3
4
5
Perfectly
53
Q

Subsequent management for severe or life threatening asthma exacerbation

A

f the patient does not respond to the above:

  • IV bronchodilator:magnesium sulphate (first-line)
  • ICU admission:if the patient is not responding to the above therapy, they may need ICU admission for further IV bronchodilation (salbutamol or aminophylline) and possible intubation
How well did you know this?
1
Not at all
2
3
4
5
Perfectly
54
Q

When to discharge a patient following a Severe asthma exacerbation

A
  • Patients can be discharged once their PEFR > 75%
  • Patients should be discharged with a course of oral steroids, inhaled steroids, and inhaled bronchodilator
  • They must have a GP appointment within 48 hours of discharge and should be given an asthma plan
How well did you know this?
1
Not at all
2
3
4
5
Perfectly
55
Q

Complications of asthma exacerbation

A
  • Intubation:those who fail to respond to medical therapy will require ICU admission and intubation
  • Death:respiratory failure can lead to death. Three people die from asthma everyday in the UK
How well did you know this?
1
Not at all
2
3
4
5
Perfectly
56
Q

Prognosis for asthma exacerbation

A

In 2017, approximately 1,500 people died from asthma in the UK. On average, 3 people die every single day from asthma. The UK has one of the highest asthma-related death rates in Europe.

How well did you know this?
1
Not at all
2
3
4
5
Perfectly
57
Q

Define Lung cancer

A

Lung cancer is the uncontrolled division of epithelial cells which line the respiratory tract.

The majority of lung cancers are primary bronchial carcinomas. These are categorised into small-cell lung cancer (SCLC) and non-small cell lung cancer (NSCLC).

How well did you know this?
1
Not at all
2
3
4
5
Perfectly
58
Q

Epidemiology of lung cancer

A
  • Lung cancer is the third most common cancer in the UK behind breast and prostate.
  • Lung cancer accounts for 35,000 deaths within the UK alone, which is more than breast and colorectal cancer combined.
  • Lung cancers are strongly associated with smoking.
  • Slightly more common in men than women but incidence in women is increasing due to women smoking habits
How well did you know this?
1
Not at all
2
3
4
5
Perfectly
59
Q

RF for lung cancer

A
  • Increasing age: adenocarcinomas are an exception, often occurring in younger patients.
  • Smoking: tobacco smoking or environmental smoke exposure.
  • Other environmental exposure: radon, asbestos, arsenic, chromium, air pollution and radiation.
  • Family history: some gene mutations are known to be associated with an increased risk of lung cancer development.
How well did you know this?
1
Not at all
2
3
4
5
Perfectly
60
Q

Pathophysiology of small cell lung cancer

A
  • 15%of lung cancer cases
  • Location: central lesion near the main bronchus
  • Derived from neuroendocrine Kulchitsky cells
  • Contain neurosecretory granules that can release neuroendocrine hormones. This makes SCLC responsible for multiple paraneoplastic syndromes:
    • SIADH → hyponatraemia
    • Ectopic ACTH→ Cushing’s syndrome
    • Lambert-Eatonmyasthenic syndrome
  • Rapid growth and patients usually present in an advanced stage
How well did you know this?
1
Not at all
2
3
4
5
Perfectly
61
Q

What % of lung cancers are NSCLC

A

85%

How well did you know this?
1
Not at all
2
3
4
5
Perfectly
62
Q

Patho of Lung Squamous cell carcinoma

A
  • Location: central lesion
  • Squamous, or square shaped, cells that produce keratin
  • Paraneoplastic syndromes:Hypertrophic pulmonary osteoarthropathy: causes inflammation of the bones and joints in the wrists and ankles, and clubbing of the fingers and toesPTHrP→ hypercalcaemia
How well did you know this?
1
Not at all
2
3
4
5
Perfectly
63
Q

Patho of Lung Adenocarcinoma

A
  • Location: peripheral lesion
  • Originate from mucus-secreting glandular cells
  • Paraneoplastic syndromes:Hypertrophic pulmonary osteoarthropathy: causes inflammation of the bones and joints in the wrists and ankles, and clubbing of the fingers and toesGynaecomastia
How well did you know this?
1
Not at all
2
3
4
5
Perfectly
64
Q

Patho of Large cell carcinoma of Lung

A
  • Location: peripheral lesion commonly, but found throughout lungs
  • Lack both glandular and squamous differentiation
  • Paraneoplastic syndromes:Hypertrophic pulmonary osteoarthropathy: causes inflammation of the bones and joints in the wrists and ankles, and clubbing of the fingers and toesEctopic β-HCG secretion
How well did you know this?
1
Not at all
2
3
4
5
Perfectly
65
Q

Pathophysiology of Lung Carcinoid tumours

A
  • Rare
  • From mature neuroendocrine cells
  • Paraneoplastic syndrome:Carcinoid syndrome which causes the secretion of hormones, particularly serotonin, which leads to increased peristalsis and diarrhoea, and bronchoconstriction causing asthma.
How well did you know this?
1
Not at all
2
3
4
5
Perfectly
66
Q

Pathophysiology of Bronchoalveolar cell tumours

A
  • Location: found throughout lungs
  • Not related to smoking
  • Can cause significant sputum production
How well did you know this?
1
Not at all
2
3
4
5
Perfectly
67
Q

Signs of lung cancer

A
  • Reduced breath sounds and a fixed monophonic wheeze may be present
  • Stony dull percussion: suggests a malignant pleural effusion
  • Supraclavicular or persistent cervical lymphadenopathy
  • Extrapulmonary manifestations:
    • Clubbing: strongly associated with squamous cell carcinoma
    • Facial plethora and swelling: due to superior vena cava obstruction
    • Hoarseness: due to recurrent laryngeal nerve palsy (Pancoast tumour)
How well did you know this?
1
Not at all
2
3
4
5
Perfectly
68
Q

Symptoms of Lung cancer

A
  • Persistent cough +/- haemoptysis
  • Dyspnoea
  • Pleuritic chest pain
  • Recurrent pneumonia
  • Constitutional symptoms as the body mounts an immune response against the cancer cells:
    • Fever
    • Weight loss and anorexia
    • Night sweats
    • Lethargy
How well did you know this?
1
Not at all
2
3
4
5
Perfectly
69
Q

Primary investigations for lung cancer

A
  • Chest X-ray: first-line
    • Hilar enlargement
    • Lung consolidation
    • “Circular opacity” – a visible lesion in the lung field
    • Pleural effusion – usually unilateral in cancer
    • Collapse
  • CT chest with contrast:gold-standard imaging; requested if there is an abnormal CXRorpersistent symptoms with a normal CXR.
  • PET-CT:if CT is suggestive of malignancy, patients should have a staging PET-CT
    • 18-fluorodeoxygenase is preferentially taken up by malignant tissue
    • PET-CT increases thediagnostic sensitivityof local and distant metastasis inNSCLC
  • Biopsy:peripheral lesionsare biopsied under image guidance (percutaneously), andcentral lesionsvia bronchoscopy and endobronchial ultrasound
How well did you know this?
1
Not at all
2
3
4
5
Perfectly
70
Q

Staging for Lung cancer

A

TNM

How well did you know this?
1
Not at all
2
3
4
5
Perfectly
71
Q

General management for Lung cancer

A
  • Smoking cessation
  • Pain management
  • Endobronchial treatment with stents or debulking can be used as part of palliative treatment to relieve bronchial obstruction caused by lung cancer.
How well did you know this?
1
Not at all
2
3
4
5
Perfectly
72
Q

Management of SCLC

A
  • Surgery not usually offered as people with SCLC usually present late with advanced disease. Surgery is only appropriate for avery smallsubset of patients with early disease (T1-2a, N0, M0).
  • Limited disease (confined to ipsilateral hemithorax):chemoradiotherapy with platinum-based agents, e.g. cisplatin
  • Extensive disease:chemoradiotherapy with platinum-based agents, or palliative chemotherapy.
How well did you know this?
1
Not at all
2
3
4
5
Perfectly
73
Q

Management of NSCLC

A
  • Non-metastatic disease (stage I-IIIa):surgery, usually with adjuvant chemotherapy
    • Typically involves lobectomy or pneumonectomy. Segmentectomy or wedge resection (taking a segment or wedge of lung to remove the tumour) is also an option.
    • Removal of lymph nodes, if affected
    • Curative radical radiotherapycan be used as an alternative to surgery
  • Metastatic disease (stage IIIb and above):palliative treatment with immunotherapy, chemotherapy, and radiotherapy
How well did you know this?
1
Not at all
2
3
4
5
Perfectly
74
Q

Complications of Lung cancer

A
  • Local obstruction:
    • Recurrent laryngeal nerve palsypresents with a hoarse voice. It is caused by the cancer pressing on the recurrent laryngeal nerve as it passes through the mediastinum.
    • Phrenic nerve palsydue to nerve compression; causes diaphragm weakness and presents as shortness of breath.
    • Superior vena cava obstruction caused by direct compression of the tumour on the superior vena cava. It presents with facial swelling, difficulty breathing and distended veins in the neck and upper chest. “Pemberton’s sign” is where raising the hands over the head causes facial congestion and cyanosis. This is a medical emergency.
    • Horner’s syndromeis a triad of partial ptosis, anhidrosis and miosis. It is caused by aPancoast’s tumour(tumour in thepulmonary apex) pressing on thesympathetic ganglion.
  • Metastatis: e.g. to hilar lymph nodes, lung pleura, heart, breasts, liver, adrenal glands, brain, and bones.
    • Adrenal: Addison’s disease
    • Liver: hepatomegaly
    • Bone: hypercalcaemia
    • Brain: focal neurological deficit
  • Paraneoplastic syndromes:
    • Syndrome of inappropriate ADH(SIADH) caused byectopic ADHsecretion by asmall cell lung cancerand presents withhyponatraemia.
    • Cushing’s syndromecan be caused byectopic ACTHsecretion by asmall cell lung cancer.
    • Hypercalcaemiacaused byectopic parathyroid hormonefrom asquamous cell carcinoma.
    • Hypertrophic pulmonary osteoarthropathy: causes inflammation of the bones and joints in the wrists and ankles, and clubbing of the fingers and toes
    • Limbic encephalitis: small cell lung cancer causes the immune system to make antibodies to tissues in the brain, specifically the limbic system, causing inflammation in these areas. This causes symptoms such as short term memory impairment, hallucinations, confusion and seizures. It is associated with anti-Hu antibodies.
    • Lambert-Eaton myasthenic syndrome: small cell carcinoma prompts the body to produce autoantibodies which bind and destroy neurons. This leads to weakness, particularly in the proximal muscles but can also affect intraocular muscles causing diplopia (double vision), levator muscles in the eyelid causing ptosis and pharyngeal muscles causing slurred speech and dysphagia (difficulty swallowing).
  • Renal: nephrotic syndrome
  • Haematological:
    • Hypercoagulability: increased risk of venous thromboemboli
    • Disseminated intravascular coagulation
How well did you know this?
1
Not at all
2
3
4
5
Perfectly
75
Q

Prognosis of lung cancer

A

Prognosis for lung cancer is poor, with a 10-year survival rate of 5.5%.

SCLC has a poorer prognosis than NSCLC, as SCLC patients will likely have disseminated disease at the point of first presentation.

Large cell lung carcinomas are anaplastic, poorly differentiated tumours with a poor prognosis.

How well did you know this?
1
Not at all
2
3
4
5
Perfectly
76
Q

Define Mesothelioma

A

Malignant mesothelioma is an aggressive epithelial neoplasm arising from the lining of the lung, abdomen, pericardium, or tunica vaginalis.

How well did you know this?
1
Not at all
2
3
4
5
Perfectly
77
Q

Epidemiology of Mesothelioma

A
  • 45,221 deaths from mesothelioma were reported in the US between 1999 and 2015
  • Often presents in patients >60 years old due to a latent disease period
  • M>F
How well did you know this?
1
Not at all
2
3
4
5
Perfectly
78
Q

RF for Mesothelioma

A
  • Increasing age: often presents in patients >60 years old due to a latent disease period
  • Male gender
  • Asbestos exposure: construction/demolition work, dock/shipyard work, electricians, plumbers, painters and carpenters.Crocidolite (blue asbestos)has the highest risk.
  • Other causes: include radiotherapy, genetics, simian virus 40
How well did you know this?
1
Not at all
2
3
4
5
Perfectly
79
Q

Pathophysiology of Mesothelioma

A

Mesothelioma is an epithelial malignancy of themesothelial cells of the pleuracovering the lungs (and epithelial cells of other organs).

The primary cause of mesothelioma isasbestos exposure, with the development of the malignancy occurring 20-40 years after exposure (after a long latent period).

The asbestos fibres make their way to the mesothelium and can get tangled up with the cell’s chromosome. Asbestosis also believed to result in macrophage and neutrophil activation, consequently generating reactive oxygen and nitrogen species.

This causes DNA damage and modification in gene expression, thus increasing the risk of cancer.

Over time, small cancerous growths (mesothelial plaques) start to cover the visceral pleura over the lungs and the parietal pleura under the chest wall. These growths start to express a lot of calretinin, a calcium-binding protein, involved in regulating calcium levels within the cell.

Asbestos exposure causes a spectrum of disease, frompleural plaques andthickening toasbestosis, mesotheliomaandlung cancer.

Because asbestos fibers affect epithelial cells, they can cause mesothelioma in nearly any of the body’s internal organs, but it’s most commonly found in the lungs and abdominal organs - the liver, spleen and bowel - or in very rare cases, the pericardium lining of the heart and testes.

The lymphatics may also be invaded, causing hilar node metastases.

How well did you know this?
1
Not at all
2
3
4
5
Perfectly
80
Q

Signs of Mesothelioma

A
  • Finger clubbing
  • Reduced breath sounds
  • Stony dull percussion: suggests a pleural effusion
  • Ascites: if peritoneal disease is present
  • Signs of metastases: e.g. lymphadenopathy, hepatomegaly, bone pain/ tenderness, abdominal pain/obstruction
How well did you know this?
1
Not at all
2
3
4
5
Perfectly
81
Q

Symptoms of Mesothelioma

A
  • Shortness of breath
  • Cough
  • Pleuritic chest pain or chest wall pain
  • Bloody sputum: if blood vessels are affected
  • Constitutional symptoms:
    • Fatigue, fever, night sweats, weight loss
How well did you know this?
1
Not at all
2
3
4
5
Perfectly
82
Q

Primary investigations for Mesothelioma

A
  • CXR:unilateral pleural effusion, reduced lung volumes, pleural thickening, lower zone interstitial fibrosis for asbestos
  • Contrast-enhanced CT chest:performed following a suspicious CXR and may demonstrate**pleural thickening, pleural plaques and enlarged lymph nodes
How well did you know this?
1
Not at all
2
3
4
5
Perfectly
83
Q

Management for Operable Mesothelioma

A

Surgery

  • Extrapleural pneumonectomy
  • Pleurectomy with decortication (removing the pleural lining + tumour masses)
  • Rarely curative

+/- Chemotherapy

  • Cisplatin
  • Pemetrexed

+/- Radiotherapy

How well did you know this?
1
Not at all
2
3
4
5
Perfectly
84
Q

Management for Inoperable Mesothelioma

A

Chemotherapy

  • Cisplatin
  • Pemetrexed

+/- Radiotherapy

How well did you know this?
1
Not at all
2
3
4
5
Perfectly
85
Q

Complications of Mesothelioma

A
  • Pneumothorax: a mesothelioma can destroy the lung tissue between the bronchial tree and the pleural space, leading to air in the pleural space
  • Local invasion of structures:dysphagia; hoarseness; cord compression; Horner’s syndrome
  • Metastasis: metastases to the contralateral lung, peritoneum and brain
How well did you know this?
1
Not at all
2
3
4
5
Perfectly
86
Q

Prognosis of Mesothelioma

A

The prognosis for mesothelioma is very poor, as only 5-10% of patients live beyond 5 years after their diagnosis. The median survival is only 12 months.

How well did you know this?
1
Not at all
2
3
4
5
Perfectly
87
Q

Define Pulmonary embolism

A

Pulmonary embolism (PE) refers to obstruction of the pulmonary vasculature, secondary to an embolus.

How well did you know this?
1
Not at all
2
3
4
5
Perfectly
88
Q

RF/aetiology for PE

A

Virchow’s triad causing clots:

  • Hypercoagulability: cancer, surgery (activates clotting cascade), oestrogen (pregnancy, contraceptive pill, HRT), nephrotic syndrome, sepsis, thrombophilia (factor V leiden mutation, protein C and S deficiency, antiphospholipid antibody syndrome)
  • Venous stasis: recent surgery, DVT, immobility (long-haul travel/ hospitilisation), >60 yrs of age, obesity, other co-morbidities e.g. heart failure
  • Endothelial damage: lower limb trauma, previous VTE, venous surgery, infections, toxins e.g smoking
  • Rarer causes:
    • Right ventricular thrombus (post-MI)
    • Septic emboli (right-sided endocarditis - bacterial vegetation)
    • Fat embolism (due to long bone fracture)
    • Air embolism
    • Amniotic fluid embolism
    • Neoplastic cells
    • Parasites
    • Foreign material during IV drug misuse
How well did you know this?
1
Not at all
2
3
4
5
Perfectly
89
Q

Pathophysiology of PE

A

Emboli typically originate in the lower extremities, most commonly secondary to a deep vein thrombosis (DVT) in the calf:

Once the clot has formed, the increased pressure in the vein can cause a part of the main clot to break free, becoming a thromboembolus which can travel downstream towards the heart and gets into the right atrium, and then into the right ventricle to get pumped into the lungs where it can get lodged.

When a pulmonary embolism happens, a blockage in any of the arteries leads to a decrease in blood flow to lung tissue downstream.

If there is no blood flowing past an alveoli, then this means that the alveoli getting ventilated with fresh air but not getting perfused with blood. This is called a ventilation perfusion mismatch or a V/Q mismatch. The body needs oxygenated blood to function and can therefore only tolerate a bit of a V/Q mismatch, before the lungs are no longer able to meet the needs of the body.

After some hours, the non-perfused lung no longer produces surfactant resulting in alveolar collapse which in turn exaggerates hypoxaemia.

How well did you know this?
1
Not at all
2
3
4
5
Perfectly
90
Q

Signs of PE

A
  • Hypoxia
    • Cyanosis may be present
  • Deep vein thrombosis: swollen, tender calf
  • Pyrexia may be present
  • Tachypnoea and tachycardia
  • Crackles
  • Hypotension: SBP <90 mmHg suggests a massive PE
  • Elevated JVP: suggests cor pulmonale
  • Right parasternal heave: suggests right ventricular strain
How well did you know this?
1
Not at all
2
3
4
5
Perfectly
91
Q

Symptoms of PE

A
  • Pleuritic chest pain
  • Dyspnoea
  • Cough +/- haemoptysis
  • Fever
  • Fatigue
  • Syncope: a red flag symptom
How well did you know this?
1
Not at all
2
3
4
5
Perfectly
92
Q

Primary investigations of PE

A
  • CXR:performed to rule out alternative pathology. It is typically normal in a PE, although a wedge-shaped opacification may be seen
  • ECG:
    • Sinus tachycardia
    • RBBBandright axis deviationsuggest right heart strain
    • S1Q3T3: large S wave in lead I; large Q wave in lead III; inverted T-wave in lead III (a classic finding but only in 20% of patients)
  • CT pulmonary angiogram (CTPA): highlights the pulmonary arteries to demonstrate any blood clots.
  • D-dimer:detect fibrin breakdown products
How well did you know this?
1
Not at all
2
3
4
5
Perfectly
93
Q

Wells score for PE

A

Above 4 - High probability of PE
4 or below - Low probability of PE

Clinical signs and symptoms of a DVT - 3
PE is the number one diagnosis or equally likely - 3
Tachycardia (over 100bpm) - 1.5
Immobilised for more than 3 days or surgery in previous 4 weeks - 1.5
Previous objectively diagnosed PE or DVT - 1.5
Haemoptysis - 1
Malignancy with treatment within the last 6 months, or palliative - 1

How well did you know this?
1
Not at all
2
3
4
5
Perfectly
94
Q

What bacteria causes Whooping cough

A

Bordatella pertussis

How well did you know this?
1
Not at all
2
3
4
5
Perfectly
95
Q

How to treat whooping cough

A

Clarithromycin

How well did you know this?
1
Not at all
2
3
4
5
Perfectly
96
Q

Prevention for PE

A
  • Compression stockings
  • Frequent calf exercises during long periods of sitting still
  • Prophylactic treatment with low molecular weight heparin
How well did you know this?
1
Not at all
2
3
4
5
Perfectly
97
Q

Supportive management for PE

A
  • Admission to hospital
  • Oxygen, as required
  • Analgesia, if required
  • Adequate monitoring for any deterioration
How well did you know this?
1
Not at all
2
3
4
5
Perfectly
98
Q

Management for Massive PE

A
  • Thrombolysis e.g. alteplase: injecting a fibrinolytic medication that rapidly dissolves clots. Can be done:
    • Intravenouslyusing a peripheral cannula.
    • Directly into thepulmonary arteriesusing a central catheter.
How well did you know this?
1
Not at all
2
3
4
5
Perfectly
99
Q

Management for Non-massive PE

A

Anticoagulation:

  • Provoked:consider stopping anticoagulation at 3 months
  • Unprovoked:consider continuing anticoagulationbeyond3 months.
  • Interim anticoagulation: if a PE islikely(Well’s score >4) and investigations cannot be performed immediately, offer interim anticoagulation; if a PE isunlikely(Well’s score ≤4) and a D-dimer cannot be obtained within 4 hours, offer interim anticoagulation
How well did you know this?
1
Not at all
2
3
4
5
Perfectly
100
Q

Further management for PE

A
  • Inferior vena cava filter:to stop clots from potentially moving into the heart and then the lungs
  • Surgical embolectomy:performed if thrombolysis is contraindicated or has failed
    • Percutaneous catheter-directed thrombolysisis an alternative
How well did you know this?
1
Not at all
2
3
4
5
Perfectly
101
Q

Complications of PE

A
  • Cor pulmonale:pulmonary vasculature obstruction can lead to pulmonary hypertension with subsequent right heart strain
  • Pulmonary infarction:obstruction of the pulmonary vasculature can result in tissue necrosis
  • Sudden death: if a large pulmonary thromboembolism happens at the pulmonary saddle, then it blocks blood from going to both lungs and can cause sudden death
  • Respiratory alkalosis: hyperventilation as a response to the embolism causes rapid release of CO2 which can make the blood more alkali
  • Embolic stroke: if patient has an atrial septal defect, embolus may enter left atrium and then left ventricle and travel to other parts of the body, including the brain
  • Heparin-associated thrombocytopaenia:a side-effect of heparin therapy
How well did you know this?
1
Not at all
2
3
4
5
Perfectly
102
Q

Prognosis for PE

A

Less than 5 to 10% of symptomatic PEs are fatal within the first hour of symptoms. Haemodynamically stable patients have a lower mortality rate compared to those who present with cardiorespiratory arrest, which is associated with a very poor prognosis. Ultimately, the overall mortality at 3 months for an acute PE is 17%

How well did you know this?
1
Not at all
2
3
4
5
Perfectly
103
Q

Define TB

A

Tuberculosis (TB) is a granulomatous disease caused by Mycobacterium tuberculosis.

How well did you know this?
1
Not at all
2
3
4
5
Perfectly
104
Q

Epidemiology of TB

A
  • 1.7 billion people worldwide have latent TB
  • Common in South Asia and sub-Saharan Africa
  • Prevalent in immunocompromised individuals
How well did you know this?
1
Not at all
2
3
4
5
Perfectly
105
Q

RF for TB

A
  • Contactwith a person with active TB
  • Endemic regions:South Asia or sub-Saharan Africa
  • Homelessness
  • Alcohol or drug abuse
  • Immunocompromised: e.g. secondary to HIV, steroid use, malnutrition, immunosuppression medication
  • Silicosis:impairs macrophage function
  • Haematological malignancy
How well did you know this?
1
Not at all
2
3
4
5
Perfectly
106
Q

General pathophysiology of TB

A

Macrophages struggle to clear M. tuberculosis due to its waxy mycolic acid capsule which confers protection (the waxy membrane also prevents binding with normal stains - known as acid fastness).

The TB bacteria are very slow dividing with high oxygen demands (aerobes). It spreads via respiratory droplets from patients with active disease. After primary infection, immunocompetent patients can harbour the infection and remain asymptomatic (latent TB). In immunocompromised patients, reactivation and failure to contain the bacteria can manifest as secondary TB. It can then spread systemically, resulting in miliary TB.

How well did you know this?
1
Not at all
2
3
4
5
Perfectly
107
Q

Difference between primary, latent, secondary and miliary TB

A

Primary - first exposure, often asymptomatic

Latent - after primary, bacteria dormant, resulting in negative sputum culture but positive Mantoux, not infectious

Secondary - Immunocompromised patients maybe develop secondary when latent reactivates, infectious, occurs in lung apex, bacteria can spread locally to form a caseating granulomata or systemically (miliary)

Miliary - due to lympho-haematogenous spread to multiple organs e.g. heart, lungs, spleen, liver etc

How well did you know this?
1
Not at all
2
3
4
5
Perfectly
108
Q

Signs of TB

A
  • Auscultation: often normal; crackles may be present
  • Clubbing: if long-standing
How well did you know this?
1
Not at all
2
3
4
5
Perfectly
109
Q

Symptoms of TB

A
  • Cough +/- haemoptysis
  • Dyspnoea
  • May be chest pain present
  • Systemic symptoms:
    • Fever
    • Lethargy
    • Weight loss
    • Night sweats
    • Lymphadenopathy
How well did you know this?
1
Not at all
2
3
4
5
Perfectly
110
Q

Explain Mantoux screening

A
  • Used to look for a previous immune response to TB: indicates possible previous vaccination, latent or active TB.
  • Intradermal injection tuberculin
  • A type IV hypersensitivity reaction takes place
  • This reaction is measured as a diameter ofindurationthat occurs across the forearm
  • If positive, assess for active disease
How well did you know this?
1
Not at all
2
3
4
5
Perfectly
111
Q

Explain Interferon-gamma release assay (IGRA)

A
  • Involves taking a sample of blood and mixing it with antigens from the TB bacteria. In a person that has had previous contact with TB the white blood cells have become sensitised to those antigens and will release interferon-gamma.
  • More sensitive than the Mantoux test
  • Used in various situations, such as:
    • If the Mantoux test is positive or inconclusive
    • If the Mantoux test may be falsely negative
How well did you know this?
1
Not at all
2
3
4
5
Perfectly
112
Q

How to investigate Active TB

A
  • CXR
    • Primary TBmay show patchy consolidation, pleural effusions and hilar lymphadenopathy
    • Latent disease may show ghon complex
    • Reactivated TBmay show patchy or nodular consolidation with cavitation (gas filled spaces in the lungs) typically in the upper zones
    • DisseminatedMiliary TB show “millet seeds” uniformly distributed throughout the lung fields
  • Microbiology:send three deep cough sputum samples; analyse with Ziehl-Neelsen stain (will turn red) and Mycobacterium culture
    • Bronchoscopy with lavage if sputum can’t be obtained
    • Lymph node aspiration for biopsy
  • Nucleic-Acid Amplification Test (NAAT): rapid diagnostic test conducted on sputum or urine if specific criteria are met e.g. co-existing HIV, risk of multi-drug resistance or, aged < 15 years
  • HIV and hepatitis status:assess for co-infection
How well did you know this?
1
Not at all
2
3
4
5
Perfectly
113
Q

Prevention of TB

A

BCG vaccine: intradermal injection of live attenuated (weakened) TB.

How well did you know this?
1
Not at all
2
3
4
5
Perfectly
114
Q

Management for latent TB

A
  • Isoniazid andrifampicin for 3 months: to people younger than 35 years old if hepatotoxicity is a concern

OR

  • Isoniazid only for 6 months: if interactions with rifampicin are a concern
  • Pyridoxine (vitamin B6): usually co-prescribed with isoniazid prophylactically to help prevent peripheral neuropathy
  • Directly observed therapy:
    • Offered as a three times a week dosing regimen
    • Involves a key worker observing the person swallow each dose of medication
    • Video observed therapy may also be an option
How well did you know this?
1
Not at all
2
3
4
5
Perfectly
115
Q

Side effect of Isoniazid

A

Peripheral neuropathy

How well did you know this?
1
Not at all
2
3
4
5
Perfectly
116
Q

Side effect of Rifampicin

A

Orange or red tears or urine

How well did you know this?
1
Not at all
2
3
4
5
Perfectly
117
Q

Management of Active TB

A
  • Notification: all cases of active TB need to be notified within three working days
  • Contact tracing: test contacts
  • Isolate patients: negative pressure rooms are used to prevent airborne spread
  • Initial phase:rifampicin, isoniazid, pyrazinamide and ethambutol (RIPE) for two months
  • Continuation phase:rifampicin and isoniazid for a further four months
  • Multi-drug resistant (MDR) TB: treatment will be extended for 1-24 months, with at least six drugs
  • Pyridoxine (vitamin B6): usually co-prescribed with isoniazid prophylactically to help prevent peripheral neuropathy
  • Directly observed therapy:
    • Offered as a three times a week dosing regimen
    • Involves a key worker observing the person swallow each dose of medication
    • Video observed therapy may also be an option
How well did you know this?
1
Not at all
2
3
4
5
Perfectly
118
Q

Complications of TB

A
  • Empyema
  • Aspergilloma
  • Bronchiectasis
  • Pneumothorax: TB is a cause of secondary spontaneous pneumothorax
  • Miliary TB:massive lymphohaematogenous spread causing multi-organ involvement
  • Extra-pulmonary disease: CNS (TB meningitis), vertebral bodies (Pott’s disease), adrenals (Addison’s disease), cervical lymph nodes (scrofuloderma), renal and GI tract
  • Drug-related side effects: refer to ‘other notes’ below
How well did you know this?
1
Not at all
2
3
4
5
Perfectly
119
Q

Prognosis for TB

A

TB is a treatable condition, with a mortality rate of approximately 5%.

Without treatment, the mortality rate can be greater than 50%.

Prognosis is significantly worse for immunocompromised patients and those with extra-pulmonary disease.

How well did you know this?
1
Not at all
2
3
4
5
Perfectly
120
Q

How to mitigate Isoniazid side effects

A

Prescribe pyridoxine (Vit B6) in combination

How well did you know this?
1
Not at all
2
3
4
5
Perfectly
121
Q

Side effects of Pyrazinamide

A

Hepatitis, Gout, Arthralgia and myalgia

How well did you know this?
1
Not at all
2
3
4
5
Perfectly
122
Q

Side effects of Ethambutol

A

Optic neuritis

How well did you know this?
1
Not at all
2
3
4
5
Perfectly
123
Q

Define Pneumonia

A

Pneumonia is an acute inflammation of the terminal bronchioles and the area surrounding the alveoli.

How well did you know this?
1
Not at all
2
3
4
5
Perfectly
124
Q

Aetiology and types of pneumonia

A
  • Bacterial
    • Streptococcus pneumoniae
    • Haemophilus influenzae
    • Staphylococcus aureus
    • Klebsiella pneumonia
    • Moraxella catarrhalis
    • Pseudomonas aeruginosa
    • Mycobacterium tuberculosis
    • MRSA
    • Causes of atypical pneumonia: mycoplasma pneumoniae, chlamydophila pneumoniae, legionella pneumophila, coxiella burnetti, chlamydia psittaci
  • Viral
    • Respiratory syncytial virus (RSV)
    • Influenza
  • Fungal
    • Pneumocystic jirovicii
  • Idiopathic interstitial pneumonia: group of non-infective causes e.g. cryptogenic organising pneumonia which may occur as a complication of rheumatoid arthritis or amiodarone use.
How well did you know this?
1
Not at all
2
3
4
5
Perfectly
125
Q

RF for Pneumonia

A
  • Extremes of age: young children and the elderly are particularly at risk
  • Preceding viral infection
  • Immunosuppressed: e.g. due to steroid use
  • Intravenous drug abuse:Staphylococcus aureus
  • Respiratory conditions: asthma, COPD, malignancy, cystic fibrosis
126
Q

Pathophysiology of Pneumonia

A

Whilst the term ‘pneumonia’ technically refers to any inflammatory reaction affecting the alveoli, it is most commonly secondary to infection. The inflammation brings water into the lung tissue, which makes it harder to breathe. It is most commonly caused by bacteria, but may be caused by viruses and fungi. There may also be non-infective causes of pneumonia.

Sometimes microbes can enter and evade the body’s defences. These microbes typically multiply and cross over from the airways into the lung tissue, creating an inflammatory response.

The tissue fills with white blood cells as well as proteins, fluid, and red blood cells if a nearby capillary is damaged in the process.

Pneumonia can be categorised by how it’s acquired:

  • Community acquired pneumonia (CAP): pneumonia acquired outside a hospital setting
  • Hospital-acquired pneumonia (HAP): pneumonia that develops more than 48h after hospital admission

Other types of pneumonia include:

  • Aspiration pneumonia: due to foreign material lodging in the lungs. Microbes on the foreign material can cause infection. Aspiration pneumonia can also happen with drinks, or vomiting of gastric contents.
  • Atypical pneumonia: pneumonia caused by an organism that cannot be cultured in the normal way or detected using a gram stain. They don’t respond to penicillins and can be treated with macrolides (e.g. clarithomycin), fluoroquinolones (e.g. levofloxacin) or tetracyclines (e.g. doxycycline).

Another way we can characterise pneumoniais by where the infection is:

  • Bronchopneumonia: infection can be throughout the lungs involving the bronchioles as well as the alveoli.
  • Atypical or interstitial pneumonia: infection is mainly just outside the alveoli in the interstitium.
  • Lobar pneumonia: infection causes complete consolidation of a whole lobe of the lung.
127
Q

Signs of Pneumonia

A
  • Reduced breath sounds
  • Bronchial breathing: harsh breath sounds equally loud on inspiration and expiration
  • Coarse crepitations
  • Dullness to percussion due to lung tissue collapse and/or consolidation.
  • Hypoxia
  • Tachycardia
  • Tachypnoea
  • Pyrexia
128
Q

Symptoms of Pneumonia

A
  • Productive cough +/- haemoptysis
  • Pleuritic chest pain
  • Dyspnoea
  • Fever
  • Night sweats
  • Fatigue
  • Delirium: acute confusion associated with infection
  • Atypical pneumonia: dry cough, mild dyspnoea, flu-like symptoms, and mild or no fever.
129
Q

Primary investigations for Pneumonia

A
  • CXR:consolidation caused by inflammatory exudate within alveoli and bronchioles
    • Atypical pneumoniacauses interstitial inflammation instead, so CXR may be normal
    • Can also identify underlying malignancy
  • FBC:leukocytosis
  • U&Es:deranged in severe disease
  • CRP:raised
  • ABG:perform if hypoxic to assess for respiratory failure
  • Sputum culture:allows assessment of organism and antibiotic sensitivities
130
Q

Explain CURB-65

A

One point for each of these:

Confusion

Urea: Above 7 mmol/L

Resp rate: 30/min or above

BP: SBP below 90mmHg or DBP below 60mmHg

65 y/o or above

131
Q

Use of CURB-65 in Secondary care

A

Consider community based care if CURB score 0 or 1

Consider hospital based care if score is 2

Consider ICU if 3 or above

132
Q

General management for Pneumonia

A

O2, Analgesia and Antibiotics

133
Q

Management for Community acquired Pneumonia

A
  • Low severity (CURB ≤ 1):oral amoxicillinORdoxycycline/clarithromycin if penicillin-allergic or an atypical pathogen is suspected; usually a 5 day course
  • Moderate severity (CURB 2): amoxicillin;addclarithromycin if an atypical pathogen is suspected; usually a 5 day course
  • High severity (CURB ≥ 3):IV co-amoxiclavandclarithromycin are often used
134
Q

Management for hospital acquired Pneumonia

A
  • Low severity:oral co-amoxiclav
  • High severity:a**broad-spectrum antibiotic, such as IV tazocin or ceftriaxone
  • Suspected or confirmed MRSA:addvancomycin
135
Q

Complications of Pneumonia

A
  • Acute respiratory distress syndrome:associated with a 30-50% mortality rate and usually requires mechanical ventilation
  • Sepsis:complicates severe CAP and may be fatal, particularly in immunocompromised patients
  • Lung abscess:may require prolonged antibiotic therapy and drainage; can occur due toKlebsiellaorStaphylococcalpneumonia
  • Pleural effusion:parapneumonic effusions can either be sterile or infected (empyema)
136
Q

Define Pneumocystis Pneumonia

A

Pneumocystis pneumonia (PCP) is an opportunistic respiratory infection caused by the fungus, Pneumocystis jirovecii.

137
Q

RF for PCP

A
  • HIV/AIDS: PCP is associated with a CD4 count < 200/mm^3
  • Primary immunodeficiency conditions
  • Secondary immunodeficiency: e.g. steroids
  • Other causes of immunosuppression: e.g. haematological malignancies
138
Q

Signs of PCP

A
  • Chest examination is often normal
  • Extrapulmonary manifestations(rare):
    • Lymphadenopathy
    • Hepatosplenomegaly
    • Choroid lesions: benign naevi at the back of the eye
  • Pyrexia
139
Q

Symptoms of PCP

A
  • Dyspnoea: characteristically exertional
  • Dry cough
  • Fever
  • Night sweats
  • History of immunosuppression
140
Q

Primary investigations for PCP

A
  • Oxygen saturation:patients with PCP characteristically desaturate onexertion
  • Arterial blood gas:type 1 respiratory failure
  • Chest X-ray:may reveal bilateral interstitial infiltrates but can be normal
  • Induced sputum:silver staining to identify PCP
141
Q

Treatment of PCP

A
  • Trimethoprim/sulfamethoxazole (co-trimoxazole):first-line therapy
  • Prednisolone:indicated if hypoxic with pO2< 9.3 kPa, to reduce the risk of respiratory failure (< 50% risk) and death
  • IV/ nebulised pentamidine:this is reserved for severe cases where co-trimoxazole is contraindicated or has failed
142
Q

Complications of PCP

A
  • Pneumothorax: lung destruction can lead to the formation of cysts (pneumatoceles) and secondary pneumothoraces
  • Respiratory failure: usually causes type 1 respiratory failure
  • Immune reconstitution inflammatory syndrome (IRIS):as the immune system begins to recover with antiretroviral therapy, T cells mount an aggressive immune response against previously acquired infections, thus causing a paradoxical worsening of symptoms
143
Q

Define Idiopathic Pulmonary Fibrosis (IPF)

A

Pulmonary fibrosis describes interstitial fibrosis of the lung parenchyma and has a number of causes.

Idiopathic pulmonary fibrosis (IPF) is the most common cause of interstitial fibrosis and has an unknown aetiology.

144
Q

Epidemiology of IPF

A
  • IPF is the most common interstitial lung disease. It has an incidence of 7.44 per 100,000 people
  • M>F
  • More common in the elderly
145
Q

Aetiology of Pulmonary fibrosis

A

Most commonly Idiopathic

Other causes:
Upper zone PF:
- Coal workers pneumoconiosis
-Silicosis
-Hypersensitivity
- Ankylosing spondylitis
- CF
- Sarcoidosis
- TB

Lower zone PF:
- Idiopathic
- Asbestosis
- Drug induced: Amiodarone, bleomycin, methotraxate, nitrofurantoin
- SLE
- Radiation

146
Q

RF for IPF

A
  • Advanced age:the mean age at diagnosis is 60-70 years of age
  • Male gender: twice as common in men
  • Smoking
  • Family history
  • Dust exposure:raising birds, metal, wood
147
Q

Pathophysiology of IPF

A

Physiology:

When the alveolar lining is damaged, type I pneumocytes release transforming growth factor beta 1, which gets the type II pneumocytes to stimulate fibroblasts to proliferate and develop into myofibroblasts (fibroblasts with some smooth muscle cell properties).

The myofibroblasts secrete reticular fibers, a type of collagen which provides structural strength, as well as elastic fibers, which provide the rubber-band like elasticity of the lungs.

The myofibroblasts then undergo apoptosis, or programmed cell death.

Pathology:

Idiopathic pulmonary fibrosis is the ongoing repair process of having excess collagen or scar tissue in the interstitial tissue of the lung.

Although the aetiology is unknown, different stimuli e.g. smoking have been speculated to induce a pro-inflammatory and pro-fibrotic response. The new hypothesis suggests a complex interaction between the epithelium and fibroblasts.

The fundamental problem is that type II pneumocytes over-proliferate during the repair process and it leads to too many myofibroblasts and too much collagen.

The myofibroblasts don’t undergo apoptosis and instead continue to make even more collagen. As the collagen accumulates, it thickens the interstitial layer between the alveoli and the capillary and that leads to problems with ventilation - carbon dioxide leaving the body - and oxygenation - oxygen getting into the body.

The excess collagen also causes the lungs to become stiff, making it harder for air to flow in and out:

It causes a decrease in total lung capacity, forced vital capacity, and the forced expiratory volume in 1 second.

The excess collagen also leads to the loss of alveoli, creating cysts, surrounded by thick walls in a pattern called “honeycombing”.

148
Q

Signs of IPF

A
  • Bibasal fine end-inspiratory crackles, predominantly in lower zones
  • Clubbing
  • Cyanosis
149
Q

Symptoms of IPF

A
  • Progressive dyspnoea
  • Non-productive cough
  • Malaise
150
Q

Primary investigations for IPF

A
  • CXR:bilateral reticulonodular shadowing - irregular, peripheral opacities (‘ground-glass’ then later progressing to ‘honeycombing’) and mainly affecting the lower zones
  • High-resolution CT thorax:the investigation of choice to confirm the diagnosis, demonstrating increased reticulation and honeycombing, mainly in the lower zones
  • Spirometry: arestrictivepattern is seen
    • Impaired gas exchange: reduced transfer factor (TLCO)
151
Q

Management for IPF

A
  • Supportive care:
    • Pulmonary rehabilitation
    • If the patient is breathless on exertion, then consider ambulatory oxygen therapy and/or long-term oxygen therapy
    • Vaccinate againstpneumococcusandinfluenza
  • Anti-fibrotic agents:
    • Pirfenidoneornintedanibare indicated if FVC is 50% - 80% of predicted
  • Lung transplantation:
    • May be considered if there are no contraindications and the patient can tolerate surgery
152
Q

Complications of IPF

A
  • Lung cancer:there is an increased risk of bronchogenic carcinoma
  • Pulmonary hypertension and cor pulmonale
153
Q

Define Sarcoidosis

A

Sarcoidosis is a granulomatous inflammatory multi-systemic disease in which any organ can be affected, although the lungs are the predominantly affected organ.

154
Q

Epidemiology of Sarcoidosis

A
  • Sarcoidosis has an incidence of 1-35.5/100,000
  • Common in Afro-Caribbeans and Scandinavians
  • F>M
  • There are two spikes in incidence, in young adulthood and again around age 60.
155
Q

RF for Sarcoidosis

A
  • Afro-Caribbean and Scandinavian ethnicity
  • Young adults: commonly presents at 20-40 years of age
  • Female gender
  • Family history
156
Q

Pathophysiology of Sarcoidosis

A

Sarcoidosis is an inflammatory condition defined by the presence of non-caseating granulomas, which are nodules of inflammation full of macrophages.

Although the aetiology is unknown, it is thought to be due to a type IV hypersensitivity reaction against an unknown antigen. This reaction is uncontrolled.

A T-cell-mediated immune response to an antigenic stimulus attracts other immune cells and causes the formation of granulomas (small nodules with T-cells on the periphery and macrophages in the centre). The granulomas in sarcoidosis are non-caseating which means that there is no tissue necrosis at the centre of the granuloma.

Oftentimes, macrophages fuse together to form a single large multi-nucleated cell called a Langhans giant cells.

The macrophages begin to release local mediators that result in inflammation.

It can involve nearly every organ, but they most often involves hilar lymph nodes which are lymph nodes that are near the point where the bronchi meets the lung.

157
Q

Signs of Sarcoidosis

A
  • Cervical and submandibular lymphadenopathy
  • Lupus pernio: a lupus-type rash
  • Erythema nodosum: dusky coloured nodules on the shins
158
Q

Symptoms of Sarcoidosis

A
  • Cough: non-productive
  • Dyspnoea: gradual onset
  • Polyarthralgia
  • Uveitis:
    • Red-eye
    • Photophobia
  • Constitutional symptoms: swinging fever, fatigue, weight loss
159
Q

Presentation of acute v chronic sarcoidosis

A

Acute sarcoidosis usually presents with features such as a swinging fever, polyarthralgia and erythema nodosum. In contrast, insidious sarcoidosis is commonly associated with a non-productive cough, dyspnoea and fatigue.

160
Q

Syndromes associated with Sarcoidosis

A
  • Lofgren’s syndrome: an acute form of sarcoidosis associated with migratory polyarthritis, erythema nodosum, fever, and bilateral hilar lymphadenopathy; has a very good prognosis
  • Heerfordt’s syndrome: causes facial nerve palsy, fever, uveitis and parotitis
  • Mikulicz’s disease: bilateral parotid and lacrimal gland enlargement; can also occur due to TB and lymphoma.
161
Q

Primary investigations for Sarcoidosis

A
  • Mainly a clinical diagnosis
  • Primary investigations
    • Routine bloods:inflammatory markers may be raised, can screen for other organ involvement
    • Serum calcium:hypercalcaemia (10%) (due to macrophages in non-caseating granulomas activating vitamin D)
    • Angiotensin-converting enzyme (ACE):elevated (but with poor sensitivity (60%) and specificity (70%))
    • Serum soluble interleukin-2 receptor: raised
    • CXR:first-line imaging; may show hilar lymphadenopathy or bilateral infiltrates
162
Q

Gold standard investigation for Sarcoidosis

A

Tissue biopsy: usually done by doing bronchoscopy with ultrasound guided biopsy of mediastinal lymph nodes. Shows characteristic non-caseating granulomas with epithelioid cells.

163
Q

Differentials for Sarcoidosis

A
  • Tuberculosis
  • Lymphoma
  • Hypersensitivity pneumonitis
  • HIV
  • Toxoplasmosis
  • Histoplasmosis
164
Q

Management of pulmonary sarcoidosis

A
  • Treatment either requires observation or steroids
  • Pulmonary disease
    • Asymptomatic non-progressive disease:observation
    • Symptomatic or progressive disease:
      • First-line:corticosteroids, e.g. inhaled budesonide or oral prednisolone
      • Second-line: immunosuppressants, e.g. methotrexate or azathioprine
      • End-stage: considerlung transplantation
    • Acute respiratory failure: oral or IV corticosteroid and ventilatory support
165
Q

Complications of Sarcoidosis

A
  • Respiratory:pulmonary hypertension, respiratory failure, fibrosis(upper zone)
  • Cardiovascular:cor pulmonale, heart block
  • Liver: cirrhosis, cholestasis
  • Kidney: kidney stones, nephrocalcinosis, interstitial nephritis
  • Bones: arthralgia, arthritis, myopathy
  • Central nervous system:encephalopathy, nerve palsies e.g. facial nerve, meningeal disease
  • Ocular: keratoconjunctivitis sicca (dryness of conjunctiva and cornea), uveitis, optic neuritis
166
Q

Define Bronchiectasis

A

Bronchiectasis is the permanent dilation of bronchi due to the destruction of the elastic and muscular components of the bronchial wall.

167
Q

Epidemiology of Bronchiectasis

A
  • F>M
  • Present at any age but increases with age
168
Q

Pathophysiology/aetiology of Bronchiectasis

A

Bronchiectasis results from diseases that cause chronic inflammation.

  • Primary ciliary dyskinesia: cilia don’t move normally which leaves mucus stuck in the airways. Bacteria trapped in the mucus start to multiply and can cause a pneumonia. If that happens repeatedly, it results in chronic inflammation.
  • Cystic fibrosis: mucus is sticky and therefore hard to sweep. The mucus accumulates and recurrent pneumonias lead to chronic inflammation
  • Deficiency of bronchial wall elements
  • Airway obstruction: e.g. tumour in the airway or outside of the airway or a foreign object that is lodged in the bronchioles. The blockage prevents the mucociliary escalator from clearing out the mucus, and leads to recurrent pneumonias and chronic inflammation.
  • Post-infections: e.g. TB, measles, bronchiolitis, pneumonia, HIV
  • Certain infections e.g. aspergillosis can worsen the problem as they can cause a hypersensitivity response which results in further inflammation of the airways.

Over time, immune cells and their cytokines can damage the ciliated epithelial cells and destroy the elastin fibres in the walls of the airway. The airways become dilated and clogged with mucus.

Bronchiectasis is an obstructive lung disease, as the inflammation causes mucus plugs to form in the airways which can obstruct airflow.

Fibroblasts move in to try and repair the damage by depositing collagen. The loss of elastin and the buildup of collagen, makes the lungs less elastic and more stiff. The stiff, mucus-filled lungs make it tough for air to flow smoothly.

The inflammation can also extend to involve the pleura.

Over time, as lung function declines, there can be hypoxia.

To adapt, the pulmonary arterioles, start to constrict - effectively diverting blood away from the most damaged areas of the lung.

But if the damage is widespread, this leads to widespread vasoconstriction of pulmonary arterioles, which leads to pulmonary hypertension.

This can make it hard for the right ventricle to pump out blood – and lead to right ventricular hypertrophy - cor pulmonale.

169
Q

Clinical manifestations of Bronchiectasis

A

Usually affects the lower lobes

  • Inspiratory crepitations
  • Wheezing
  • Productive coughing
  • Large amounts of khaki coloured sputum (sometimes flecked with blood)
  • Shortness of breath
  • Foul smelling mucus
  • Chest pain
  • Digital clubbing: due to long term hypoxia
170
Q

Investigations for Bronchiectasis

A
  • CXR/ CT scan: shows thickened and dilated bronchi and bronchioles.
  • Sputum culture: to see bacterial colonisation status
  • Pulmonary function testing e.g. spirometry: decrease in lung capacity and the ability to force air out of the lungs, as a result of decreased lung elasticity.
  • Further/ genetic testing: can be done to look for underlying conditions e.g. primary ciliary dyskinesia or cystic fibrosis.
171
Q

Management of Bronchiectasis

A
  • Antibiotics: for recurrent infections
  • Postural drainage: to remove excess mucus
  • Chest physio
  • Mucolytics
  • Bronchodilators e.g. nebulised salbutamol: useful for asthma or COPD sufferers
  • Anti-inflammatory agents e.g. long term azithromycin can reduce exacerbation frequency
  • Surgery: to remove physical obstruction e.g. foreign object
172
Q

What is the name for a collection of air around the heart

A

Pneumomediastinum

173
Q

What is the name for a collection of infected fluid in the pleural cavity

A

Empymema

174
Q

Complications of Bronchiectasis

A
  • Cor pulmonale: pulmonary hypertension leads to issues with the right ventricle ejecting blood causing the ventricle to hypertrophy
  • Pneumonia
  • Pleural effusion
  • Pneumothorax
  • Haemoptysis
175
Q

Define Cystic Fibrosis

A

Cystic fibrosis (CF) is an inherited, autosomal recessive, multi-system disease affecting mucus glands.

Respiratory problems most prominent, as well as pancreatic insufficiency.

176
Q

Epidemiology of CF

A

Cystic fibrosis is the most common inherited condition in the Caucasian population, affecting 1/2500 births, whilst 1/25 of the population are carriers

177
Q

RF for CF

A
  • Family history of cystic fibrosis
  • Known parental carriers: if both parents are known carriers, the child has a 1 in 4 chance of having cystic fibrosis
  • Caucasian ethnicity
178
Q

Pathophysiology of CF

A

CF occurs due to mutations in the CF transmembrane conductance regulator (CFTR) on chromosome 7. Δ-F508 is the most common mutation, where the codon for phenylalanine (F) in the CFTR gene is deleted, resulting in proteolytic degradation.

The CFTR protein is a channel protein that pumps chloride ions into various secretions, those chloride ions help draw water into the secretions, which ends up thinning them out.

This CFTR protein with the ∆F508 mutation gets misfolded and can’t migrate from the endoplasmic reticulum to the cell membrane, meaning there’s a lack of CFTR protein on the epithelial surface, and this means that it can’t pump chloride ions out, which means water doesn’t get drawn in, and the secretions are left overly thick.

179
Q

Signs of CF

A
  • Low weight or height on growth charts
  • Nasal polyps
  • Finger clubbing
  • Crackles and wheezes on auscultation
  • Abdominal distention
180
Q

Symptoms of CF

A
  • Chronic cough
    • Can be haemoptysis if inflammation erodes into a blood vessel
  • Thick sputum production
  • Recurrent respiratory tract infections
  • Loose, greasy stools (steatorrhoea) due to a lack of fat digesting lipase enzymes
  • Abdominal pain and bloating
  • Parents may report the child tastes particularlysaltywhen they kiss them, due to the concentrated salt in the sweat
  • Poor weight and height gain (failure to thrive)
181
Q

Antenatal presentation of CF

A

Hyperechogenic bowel (appears brighter than usual) on ultrasound

182
Q

Neonatal presentation of CF

A
  • Prolonged jaundice
  • Meconium ileus: first stool passed is thick and sticky and obstructs the bowel
183
Q

Childhood presentation of CF

A
  • Recurrent chest infections (40%)
  • Failure to thrive despite a voracious appetite
  • Malabsorption: diarrhoea and steatorrhea (30%)
  • Nasal polyps and chronic sinusitis
  • Delayed puberty and short stature
  • Other features: pancreatitis, rectal prolapse, portal hypertension (5-10%)
184
Q

Adulthood presentation of CF

A
  • Recurrent chest infections
  • Atypical asthma
  • Diabetes mellitus
  • Male infertility: absence of vas deferens
  • Female subfertility
185
Q

Primary investigations for CF

A
  • Guthrie test: heel-prick test for serum immunoreactive trypsinogen (released into foetal blood when there is pancreatic damage)
  • Sweat test:gold standard test; induce sweating followed by analysis of sweat to check Cl- concentration
  • Genetic testing:Genetic testing for CFTR gene mutation can be performed during pregnancy by amniocentesis or chorionic villous sampling, or as a blood test after birth. Also offered to close relatives of people with cystic fibrosis.
186
Q

Management for CF

A
  • Airway clearance techniques:minimum 2 times per day. Chest physiotherapy and postural drainage.
  • Bronchodilator: inhaledsalbutamolfor exacerbations and prior to airway clearance techniques
  • Mucoactive agents
  • Immunomodulation
  • Antibiotics
  • Vaccinations including pneumococcal, influenza and varicella
  • Lung or heart-lung transplantation: last-line for end-stage cardiorespiratory disease
187
Q

GI and Hepatobiliary management for CF

A

Dietary supplements, PPI

188
Q

Monitoring for CF

A

Patients with cystic fibrosis are managed and followed up in specialist clinics, typically every 6 months.

They require regular monitoring of their sputum for colonisation of bacteria like pseudomonas.

They also need monitoring and screening for diabetes, osteoporosis, vitamin D deficiency and liver failure.

189
Q

Prognosis for CF

A

Although there is no cure for cystic fibrosis, the life expectancy has improved from infancy to a mean age of survival of 40 years.

  • 90% of patients with CF developpancreatic insufficiency
  • 50% of adults with CF developcystic fibrosis-related diabetesand require treatment withinsulin
  • 30% of adults with CF developliver disease
  • Most males are infertile due to absent vas deferens

Lung disease is the most common cause of death in patients with cystic fibrosis, followed by transplantation complications and advanced liver disease.

190
Q

Type 1 vs Type 2 Respiratory failure

A

Type 1:
Low O2, low or normal CO2, Normal HCO3
Examples: Pneumonia, PE

Type 2:
Low O2, High CO2, Normal HCO3 if acute or high if chronic
Examples: Emphysema, Neuromuscular disease

191
Q

Define Pleural effusion

A

A pleural effusion results when fluid collects between the parietal and visceral pleural surfaces of the thorax.

This can be exudative meaning there is a high protein count (>30g/L) or transudative meaning there is a relatively lower protein count (<30g/L).

192
Q

Epidemiology of Pleural effusion

A
  • Seen in adults and less commonly in children
  • Heart failure is the most common cause of transudative pleural effusions, whilst pneumonia and malignancy are the most common causes of exudative pleural effusions.
193
Q

Pathophysiology/aetiology of pleural effusion

A

Normal pleural fluid enters from the pulmonary microvasculature and is drained by the lung lymphatics.

Diseases which increase the filtration rate from the vasculature or decrease the absorption rate to the lymphatics will result in the accumulation of pleural fluid.

  • Transudate effusion: caused by increased hydrostatic pressure or low oncotic pressure. This causes fluid to move from the capillaries into the pleural sac.Causes include:
    • Congestive heart failure: heart can’t effectively pump to rest of body and so blood is backed up and increases pressure in pulmonary microvasculature. This blood then leaves the capillaries into the pleural space
    • Hypoalbuminaemia: may be due to liver failure (lack of protein production), nephrotic syndrome (loss of protein in the urine) and malabsorption. Decrease protein content in the capillaries causes low plasma oncotic pressure and more fluid leaves to enter the pleural space
    • Hypothyroidism: an effusion may occur in patients with severe hypothyroidism (myxoedema)
    • Peritoneal dialysis: an acute massive effusion may occur within 48 hours of initiating dialysis due to dialysis fluid crossing the diaphragm
    • Meig’s syndrome: this is the triad of a benign ovarian tumour with ascites and a pleural effusion
  • Exudative effusion: caused by inflammation, infection and malignancy. The inflammation results in protein e.g. LDH leaking out of the tissues and into the pleural space.Causes include:
    • Malignancy: tumour infiltration of pleural capillaries and cytokines increase capillary permeability and thus increased leakage of fluid into the pleural space
    • Infection: e.g. pneumonia (acute lung injury increases vascular permeability which causes movement of high protein lung interstitial fluid into pleural space). Other examples include TB and subphrenic abscess.
    • Trauma
    • Pulmonary embolism
    • Pancreatitis
    • Autoimmune and connective tissue disorders: key causes include SLE, rheumatoid pleurisy and eosinophilic granulomatosis with polyangiitis
    • Dressler’s syndrome: the immune system response that leads to Dressler’s syndrome may also cause a pleural effusion
  • Lymphatic effusion (chylothorax)
194
Q

Signs of pleural effusion

A
  • Reduced chest expansion and reduced breath sounds on the affected side
  • Decreased tactile or vocal fremitus (reduced vibration of chest wall when speaking)
  • Dullness to percussion over the effusion: classically ‘stony dull’
  • Pleural friction rub (raspy breathing sound) and bronchial breathing in the most superior aspect of the pleural effusion
  • Tracheal deviation away from the effusion if effusion is massive
195
Q

Symptoms if pleural effusion

A
  • Shortness of breath
  • Cough
  • Pleuritic chest pain: usually associated with an exudate due to pleural irritation
  • Symptoms of the underlying cause, for example:
    • Peripheral oedema (heart failure)
    • Ascites (liver cirrhosis)
    • Productive cough and fever (pneumonia)
    • Weight loss (malignancy)
196
Q

Primary investigations for pleural effusion

A
  • Chest X-ray:
    • Bluntingof thecostophrenic angle (where the diaphragm meets the ribs)
    • Fluid in the lungfissures
    • Larger effusions will have ameniscus: a curving upwards where effusion meets the chest wall and mediastinum.
    • Tracheal and mediastinaldeviation:if it is a massive effusion
  • Pleural aspiration and pleural fluid analysis
    • The lateral site, above the ribs is preferred to minimise the risk of intercostal vessel trauma
197
Q

Management of infective pleural effusion

A
  • If any one of the following are present in pleural fluid analysis, chest tube drainage is required:
    • Purulent or turbid/cloudy pleural fluid
    • Positive pleural fluid Gram stain or culture
    • Pleural fluid pH <7.2
    • Loculated pleural collection
198
Q

Management of non-malignant pleural effusion

A
  • Treat the underlying cause:e.g.**loop diuretics and sodium restriction for congestive heart failure
  • Thoracentesis for symptomatic effusion:
    • Needle drainage (thoracocentesis) or chest tube drainage may be required depending on the severity of symptoms and size of the effusion
  • Recurrent effusions:
    • Pleurodesis:scarring the pleural space by slurry injection or spraying sterile talc, this causes adhesion of the visceral and parietal pleura and prevents recurrence
    • Recurrent aspiration
    • Indwelling pleural catheter:a chest tube tunnelled under the skin used for long-term repeated pleural drainage
199
Q

Management of malignant pleural effusion

A

Potential options include:

  • Observation
  • Therapeutic pleural aspiration
  • Intercostal small-bore tube drainage
  • Talc pleurodesis
200
Q

Complications of pleural effusion

A
  • Atelectasis/lobar collapse: large effusions will compress the lungs upwards.
  • Empyema:an infected pleural effusion. Pleural aspiration shows pus, acidic pH, low glucose and high LDH. It is treated by chest drain to remove the pus and antibiotics.
  • Trapped lung: prolonged inflammation causes fibroblasts to encase the lung in a fibrous peel, preventing re-inflation.
  • Re-expansion pulmonary oedema: iatrogenicfollowing rapid expansion from rapid drainage of large effusions (>1.5 L)
  • Pneumothorax:**iatrogenicsecondary to pleural fluid aspiration or chest tube insertion
201
Q

Define Pneumothorax

A

A pneumothorax is an abnormal accumulation of air within the pleural space, which is the space between the parietal and visceral pleura.

202
Q

Epidemiology of Pneumothorax

A

The incidence of a pneumothorax is 24/100,000 a year for men, and 10/100,000 a year for women

203
Q

RF for Pneumothorax

A

Catamenial pneumothorax: a pneumothorax which occurs within 72 hours before or after the start of menstruation. It is thought to be caused primarily by endometriosis of the pleura. It accounts for 3-6% of pneumothoraces in women

204
Q

Pathophysiology and RF for Primary spontaneous pneumothorax

A
  • Typical presentation: a young, tall, healthy, male presents with sudden onset breathlessness and chest pain
  • No underlying disease
  • Due to spontaneous rupture of a subpleural bleb (thin-walled air-containing spaces)
  • Risk factors:
    • Tall, slender, young (aged 20-30)
    • Smoking
    • Marfan syndrome
    • Rheumatoid arthritis
    • Family history
    • Homocystinuria (body can’t process the amino acid methionine)
    • Diving or flying
205
Q

Pathophysiology and RF for Primary spontaneous pneumothorax

A
  • Typical presentation: a young, tall, healthy, male presents with sudden onset breathlessness and chest pain
  • No underlying disease
  • Due to spontaneous rupture of a subpleural bleb (thin-walled air-containing spaces)
  • Risk factors:
    • Tall, slender, young (aged 20-30)
    • Smoking
    • Marfan syndrome
    • Rheumatoid arthritis
    • Family history
    • Homocystinuria (body can’t process the amino acid methionine)
    • Diving or flying
206
Q

Pathophysiology and RF of Secondary Pneumothorax

A
  • Typical presentation: a middle-aged patient with COPD presents with sudden onset breathlessness and chest pain
  • There are signs of underlying disease
  • Due to ruptured bleb or bullae (fluid-filled sac) secondary to lung disease
  • Risk factors:
    • Underlying lung disease e.g. COPD, asthma, lung cancer
    • TB
    • Pneumocystic jirovecii
207
Q

Pathophysiology and RF for Tension Pneumothorax

A
  • Typical presentation: a ventilated patient suddenly becomes breathless and haemodynamically unstable. This is an emergency.
  • There may or may not be underlying disease. Usually occurs in ventilated or trauma patients.
  • Air is forced to enter the thoracic cavity without any means of escape, resulting in a ‘one-way-valve’
    • This can go on to compromise cardiac function
  • Risk factors
    • Mechanical ventilation and NIV
    • Trauma
    • Iatrogenic: central line insertion, biopsy
208
Q

Signs of Pneumothorax

A
  • Tachycardia and tachypnoea
  • Cyanosis
  • Hyperresonance ipsilaterally (resonance is higher when percussed)
  • Reduced breath sounds ipsilaterally
  • Hyperexpansion ipsilaterally: associated with tension pneumothorax
  • Contralateral tracheal deviation (trachea deviates away from the area of damage) and circulatory shock: in tension pneumothorax (emergency)
  • Hypotension: in tension pneumothorax
209
Q

Symptoms of Pneumothorax

A
  • Sudden-onset pleuritic chest pain
  • Sudden-onset dyspnoea
  • Sweating: may be present
210
Q

Investigations for pneumothorax

A
  • If tension pneumothorax is suspected, don’t wait for investigations!
  • Erect CXR:first-line investigation
    • Demonstrates a visible visceral pleural edge with no lung markings peripheral to this line
    • In atension pneumothorax, there is mediastinal shift and tracheal deviation contralaterally
    • A repeat CXR isalwaysrequired following aspiration or drainage to assess efficacy
  • CT chest:gold-standard due to accurate pneumothorax size estimation, however, it is rarely performed in clinical practice
211
Q

Management for spontaneous pneumothorax

A
  • Aspiration is usually performed at the2nd intercostal space midclavicular lineon the affected side
  • Chest drain is inserted at the5th intercostal space mid-axillaryline on the affected side within the ‘triangle of safety’
  • High-flow oxygen
  • Repeat CXR
212
Q

Management for tension pneumothorax

A
  • Emergency!
  • Immediate needle decompression with the insertion of a cannula into the second intercostal space in the midclavicular line
  • High-flow oxygen
  • Chest drain after aspiration
  • Repeat CXR
213
Q

Management for iatrogenic pneumothorax

A
  • Require monitoring
  • Aspiration is preferred if treatment is required
  • Ventilated patients and those with underlying lung disease (e.g. COPD) may require a chest drain
214
Q

Surgical management for Pneumothorax

A

Surgical management is often required, particularly for recurrent pneumothoraces:

  • Open thoracotomy and pleurectomy: removal of pleura
  • Video-assisted thoracoscopic surgery (VATS) with pleurectomy and abrasion: causes adhesion between visceral and parietal pleura to prevent recurrence
  • Surgical chemical pleurodesis: adheres lung to chest wall
215
Q

Complications of pneumothorax

A
  • Re-expansion pulmonary oedema:a large pneumothorax that has been persistent for 72 hours is prone to pulmonary oedema after pleural space evacuation
  • Cardiorespiratory arrest: may occur with a tension pneumothorax due to mediastinal shift and compromise of cardiac output due to tamponade
  • Recurrence: a single primary pneumothorax increases the risk of a subsequent ipsilateral pneumothorax
216
Q

Prognosis for pneumothorax

A

Death due to a spontaneous pneumothorax is relatively rare. However, asecondary spontaneous pneumothoraxis associated with increased mortality and morbidity compared to primary spontaneous pneumothorax.

Aniatrogenic pneumothoraxis less likely to recur than a spontaneous pneumothorax.

Whilst atension pneumothoraxshould certainly be considered a medical emergency, fatal tension pneumothorax occurs in only 3.3% of cases.

Smoking cessationreduces the risk of recurrence: the lifetime risk of a pneumothorax in healthysmokingmen is 10%, compared to only 0.1% innon-smokingmen.

217
Q

Define Pulmonary hypertension

A

Pulmonary hypertension is increased resistance and pressure of blood in the pulmonary arteries.

A mean pulmonary arterial pressure that is greater than 25 mmHg.

218
Q

What are the five groups of pulmonary hypertension?

A

Group 1 - Primary pulmonary hypertension or connective tissue disease e.g. SLE

Group 2 - Left heart failure usually due to MI or systemic HTN

Group 3 - Chronic lung disease e.g. COPD

Group 4 - Pulmonary vascular disease such as PE

Group 5 - Miscellaneous causes such as sarcoidosis, glycogen storage disease and haematological disorders

219
Q

Aetiology of Group 1 - Primary pulmonary hypertension

A

There is elevated pressure in the pulmonary arterioles, but the pressure in their capillaries and pulmonary veins is still normal.

  • Some congenital heart defects can cause this e.g. a left-to-right cardiac shunt caused by a ventricular or atrial septal defect, or a patent ductus arteriosuscan result in pulmonary hypertension and eventual reversal to a right-to-left shunt - Eisenmenger’s syndrome.
  • Pulmonary hypertension can also be seen in connective tissue disorders e.g. SLE, infections e.g. HIV, thyroid disorders, and inherited genetic mutations.The process begins with damage to the endothelial cells lining the pulmonary arteries. The damaged endothelial cells release chemicals e.g. endothelin-1, serotonin, and thromboxane. These make the pulmonary arterioles constrict and cause hypertrophy of the smooth muscle surrounding them.The damaged endothelial cells also produce less nitric oxide and prostacyclin (which, if present, would make the pulmonary arterioles dilate and inhibit smooth muscle hypertrophy).
220
Q

Aetiology of Group 2 - Left heart failure pulmonary htn

A

Pulmonary blood vessels are normal and undamaged, but the left side of the heart is unable to pump efficiently.

This causes a backup of blood in the pulmonary veins and capillary beds, which can increase the pressure in the pulmonary artery.

221
Q

Aetiology of Group 3 - COPD pulmonary htn

A

This typically causes hypoxic vasoconstriction - some area in the lung is diseased and is unable to deliver oxygen to the blood. To help adapt to this, the pulmonary arterioles in that area start to constrict - and this shuttles blood away from those damaged areas of the lung, and towards healthy lung tissue.

If the problem is widespread, there’s widespread vasoconstriction of pulmonary arterioles, and that increases pulmonary vascular resistance.

This makes it hard for the right ventricle to pump out blood. To make the same amount of blood flow through the pulmonary arterioles, the right side of the heart has to generate increased pressure and this results in pulmonary hypertension.

222
Q

Aetiology of Group 4 - Pulmonary vascular disease pulmonary htn

A

There are recurrent blood clots in the pulmonary vessels. The clots can form because of an underlying clotting disorder, and can embolise or travel to the lungs.

The clots can block pulmonary vessels which increases the resistance to blood flow and can also cause endothelial cells in the vessels to release histamine and serotonin, which constricts the pulmonary arterioles.

223
Q

Pathophysiology of Pulmonary htn

A
  • Fluid can start to squeeze out of the blood vessels in the lungs and can get into the interstitial space - pulmonary oedema. This can make gas exchange difficult.
  • Makes it a lot harder for the right ventricle to pump blood and over time it hypertrophies. Eventually the muscles of the right ventricle get so bulky that their oxygen demand exceeds the oxygen supply and it can lead to right-sided heart failure.Known as cor pulmonale if caused by chronic lung disease.
  • Right heart failurecauses blood to get backed up in the venous system. And this leads to elevated jugular venous pressure, fluid buildup in the liver - causing hepatomegaly, and fluid buildup in the legs causing oedema.The left ventricle also receives less blood, and to compensate for that it has to pump harder and faster.
224
Q

Clinical manifestations of Pulmonary htn

A
  • Shortness of breath
    • If caused by left-sided heart failure, there can be orthopnea - the shortness of breath is worse while lying flat.
  • Fatigue
  • Chest pain
  • Syncope (temporary loss of consciousness caused by a sudden drop in blood pressure)
  • Tachycardia
  • Raised JVP
  • Hepatomegaly
  • Peripheral oedema
225
Q

Investigations for Pulmonary HTN

A
  • ECG:Right sided heart strain causes ECG changes:
    • Right ventricular hypertrophyseen as larger R waves on the right sided chest leads (V1-3) and S waves on the left sided chest leads (V4-6)
    • Right axis deviation
    • Right bundle branch block
  • CXR:
    • Dilated pulmonary arteries
    • Right ventricular hypertrophy
  • Raised NT-proBNP: indicates right ventricular failure
  • ECHO: can be used to estimate pulmonary artery pressure
  • Further tests for underlying cause e.g. spirometry can be done to look for chronic lung disease.
226
Q

Management for Pulmonary HTN

A
  • Supportive treatment for complications such as respiratory failure, arrhythmias, heart failure, oedema e.g. supplemental oxygen, diuretics
  • Primary pulmonary hypertensioncan be treated with:
    • IV prostanoids (e.g. epoprostenol)
    • Endothelin receptor antagonists (e.g. macitentan)
    • Phosphodiesterase-5 inhibitors**(e.g. sildenafil)
  • Secondary pulmonary hypertensionis managed by treating the underlying cause such as pulmonary embolism or SLE.
227
Q

Complications of Pulmonary HTN

A
  • Respiratory failure
  • Heart failure
  • Arrhythmias
228
Q

Define Hypersensitivity Pneumonitis

A

Hypersensitivity pneumonitis (HP), also known as extrinsic allergic alveolitis, is the result of non-IgE mediated immunological inflammation of the lungs.

229
Q

Epidemiology of HP

A

Usually a disease of adults

230
Q

Aetiology of HP

A

Hypersensitivity pneumonitis can be caused by a variety of organic antigens e.g. coffee bean dust, moldy sugarcane, bacterial spores. The resulting disease is often named for the profession at risk.

  • Bird-fanciers lungis a reaction to bird droppings
  • Farmers lungis a reaction to mouldy spores in hay
  • Mushroom workers’ lungis a reaction to specific mushroom antigens
  • Malt workers lungis a reaction to mould on barley
  • Humidifier or air conditioner lung: caused by inhaling the spores of actinomycetes that grow in the warm water reservoirs
231
Q

RF for HP

A
  • Pre-existing lung disease
  • Specific occupations e.g. farmers, cattle workers, ventilation system workers, vets and those jobs that involve working with chemicals
  • Bird keeping
  • Regular use of hot tubs
232
Q

Pathophysiology of HP

A

Hypersensitivity pneumonitis is when a person’s immune system reacts excessively to something that’s inhaled, causing lung inflammation.

The antigens are breathed in and settle in the alveolus. It is picked up by an alveolar macrophage which takes it to the nearest lymph node.

In the lymph node, it attracts CD4 T-helper cells which release cytokines to attract more immune cells. This leads to:

  • A type III hypersensitivity reaction: develops over a period of hours; activated B cells generate IgG antibodies that bind to the organic antigen in the blood stream. This can form clusters of immune complexes.The complexes can be deposited on the basement membrane and activate the complement system ending with the attraction of neutrophils to the site. These neutrophils release lysosomal enzymes and reactive oxygen species into the area which leads to inflammation and necrosis of the capillaries as well as nearby alveoli.
  • A type IV hypersensitivity reaction: develops over 2-3 days; large numbers of activated macrophages and T-cells come to the site of antigen exposure and surround it, forming a granuloma. If the antigen isn’t removed, then the immune system can cause lasting damage to the alveoli.

Chronic inflammation causes damage to elastin fibers, and results in fibroblasts entering the tissue to deposit fibrin. More fibrin, and less elastin, results in restrictive lung disease.

The scarring also results in loss of functional alveoli, which affects gas exchange.

233
Q

Clinical manifestations of acute HP

A
  • Fever
  • Rigors
  • Headache
  • Myalgia
  • Shortness of breath
  • Cough
  • Chest tightness
234
Q

Clinical manifestations of chronic HP

A

More gradual onset

  • Sustained shortness of breath
  • Cyanosis and clubbing may develop
  • Respiratory failure
235
Q

Investigations for HP

A
  • Chest x-ray: shows diffuse infiltrate
  • Lung function tests: abnormal
  • ESR: raised
  • Bronchoalveolar lavage: high number of lymphocytes and mast cells
  • Lung biopsy: small granulomas around the bronchioles, and lymphocyte infiltration in the alveolar walls
  • Identify trigger: can use inhalation challenge - expose patient to potential triggers and monitor symptoms
236
Q

Management of HP

A
  • Eliminate trigger
  • Steroids: can be used to help with symptoms
  • Give oxygen, where necessary
237
Q

Define Occupational lung disroders/pneumoconiosis

A

Pneumoconiosis describes interstitial fibrosis secondary to occupational exposure causing an inflammatory reaction.

238
Q

Epidemiology of Occupational lung disorders

A
  • M>F
  • Presents in older people
  • Asbestos is the most common cause of death relating to work in the UK
239
Q

Aetiology and types of Occupational lung disorders

A
  • Coal worker’s pneumoconiosis: exposure to carbon in coal mines
  • Silicosis: sandblasters, quarry workers and silica miners
  • Berylliosis: aerospace industry and beryllium miners
  • Asbestosis: construction workers, plumbers, and shipyard workers
  • Byssinosis: cotton mill workers

Other occupation lung disorders include:

  • Occupational asthma
  • Hypersensitivity pneumonitis
240
Q

RF for Occupational lung disorders

A
  • Male: due to prevalence in the coal-mining industry
  • Increasing age:>50 years old due to delayed onset of disease
  • Substance exposure:e.g. **coal dust, silicon, beryllium, asbestos
241
Q

Pathophysiology of occupational lung disorders

A

The severity of pneumoconiosis is related directly to the duration and extent of exposure.

When dust particles are inhaled, they reach the terminal bronchioles and are ingested by interstitial and alveolar macrophages.

Dust particles are carried by macrophages and expelled as mucus. In people exposed to these particles over a long period of time, this process is no longer functional and macrophages accumulate in alveoli, resulting in immune system activation and lung tissue damage.

Fibroblasts then arrive on scene to try and repair the damage by depositing extracellular matrix composed of collagen.

Simple pneumoconiosis:

  • The most common type of pneumoconiosis
  • Patients are usually asymptomatic but may develop progressive massive fibrosis
  • Increases the risk of chronic lung diseases such as COPD

Progressive massive fibrosis:

  • Dust exposure results in the formation of large masses of dense fibrosis, usually in the upper lobes
  • Most commonly occurs in coal workers’ pneumoconiosis and silicosis, but may also occur in berylliosis
  • Results in symptoms such as exertional breathlessness and cough, which may be productive of black sputum
  • Lung function tests usually demonstrate a mixed obstructive and restrictive picture
242
Q

General signs of occupational lung disorders

A
  • Fine crackles
  • Wheezing
  • Clubbing
243
Q

General symptoms of occupational lung disorders

A
  • Exertional dyspnoea
  • Dry cough
    • May be productive of black sputum in progressive massive fibrosis
  • Wheezing
  • Haemoptysis
  • Weight loss
244
Q

Specific symptoms for coal workers pneumoconiosis

A

Progressive massive fibrosis, upper zone fibrosis

245
Q

Specific features of Silicosis

A

Increased risk of TB, Progressive massive fibrosis, upper zone fibrosis

246
Q

Specific features of Berylliosis

A

Non-caseating granuloma, Increased risk of lung cancer, upper zone fibrosis

247
Q

Specific features of asbestosis

A

Increased risk of lung cancer and mesothelioma, pleural plaques and low zone fibrosis

248
Q

Investigations for occupational health disorders

A
  • CXR:evidence of fibrosis. Nodular opacities in the upper zones are classical of silicosis and coal workers’ lungs. In advanced disease, there is calcification of hilar lymph nodes known as eggshell calcification.
  • Spirometry:typically restrictive pattern with FEV1/FVC > 0.7 and**reduced transfer factor (TLCO - how lungs take up oxygen)
  • High-resolution CT chest:will reveal interstitial fibrosis affecting either upper or lower zone depending on aetiology
  • Rheumatoid factor and anti-nuclear antibodies: often present in the serum of patients with progressive massive fibrosis
249
Q

Staging for occupational lung disorders

A

This is determined by the appearance on chest X-ray:

  • Category 0: small rounded opacities are absent or less profuse than category 1
  • Category 1: small rounded opacities are present but few in number
  • Category 2: small rounded opacities are numerous, but normal lung markings are visible
  • Category 3: small rounded opacities are very numerous, with normal lung markings partially or totally obscured
250
Q

Management for occupational lung disorders

A

Incurable. Management is supportive:

  • Smoking cessation and avoidance of exposure
  • Pulmonary rehabilitation
  • Supplementary oxygen:hypoxic patients may benefit from long term or ambulatory oxygen
  • Corticosteroids:may be used in berylliosis
  • Lung transplantation:used in refractory end-stage disease
251
Q

Complications of occupational lung disorders

A
  • Lung cancer
  • Cor pulmonale:secondary to pulmonary hypertension**due to chronic lung disease
  • TB:silicosis patients exposed toMycobacterium tuberculosisare at an increased of TB as silicon impairs phagolysosome formation by macrophages
  • Caplan syndrome:a**combination of rheumatoid arthritis and coal workers’ lung
252
Q

Prognosis for occupational lung disorders

A

With adequate removal of exposure and smoking cessation, pneumoconiosis may not progress in certain patients.

Prognostic factors include the extent of fibrosis at presentation and degree of exposure.

253
Q

Define Goodpastures syndrome

A

Goodpasture syndrome is an autoimmune disease that primarily affects the lungs and the kidneys.

Also known as anti-GBM disease and is a rare small-vessel vasculitis.

254
Q

Epidemiology of Goodpastures

A
  • Rare in children
  • Usually occurs in individuals over 16 years old
  • In adults its more common in men
255
Q

RF for Goodpastures

A
  • HLA-DR15
  • Lymphocyte-depleting agents (e.g. alemtuzumab): loss of regulatory T cells
  • Infection
  • Smoking
  • Cocaine
  • Oxidative stress
  • Exposure to metal dust, organic solvents, or hydrocarbons
  • Alport syndrome: 5-10% of patients with Alport syndrome may develop anti-GBM disease following transplant.
256
Q

Constitutional symptoms of Goodpastures

A
  • Lethargy
  • Fever
  • Anorexia
  • Weight loss
  • Myalgia
  • Arthralgia
257
Q

Kidney symptoms of Goodpastures

A
  • Haematuria
  • Proteinuria
  • Oliguria/ anuria
  • Hypertension
258
Q

Lung symptoms of Goodpastures

A
  • Cough
  • Shortness of breath
  • Haemoptysis
  • Pulmonary haemorrhage: may be seen on imaging
259
Q

Investigations for Goodpastures

A
  • Serum antibodies
  • Biopsy: usually of the kidney; can show inflammation of basement membrane as well as lining up of antibodies against BM if fluorescent proteins are used
  • Acute renal screen: to determine the cause of any sudden deterioration in renal function
    • Urinalysis: blood and protein suggestive of intra-renal cause.
    • Protein:creatinine ratio: quantify amount of proteinuria (e.g. is it ‘nephrotic’ range?)
    • Routine bloods: FBC, U&E, LFT, Bone, CK, Coagulation, CRP
    • Special bloods: ANA, ENA, dsDNA, ANCA, Anti-GBM, Protein electrophoresis, serum free light chains, complement, immunoglobulins, virology (hepatitis C, hepatitis B, human immunodeficiency virus +/- cytomegalovirus and epstein-barr virus), cryoglobulins.
    • Imaging: renal ultrasound +/- dopplers (to exclude obstruction, thrombus or stenosis)
  • Chest radiograph +/- CT: to assess the lungs in patients
  • Pulmonary function tests: usually not required; would show a raised diffusing capacity for carbon monoxide (DLCO) due to haemorrhage and carbon monoxide binding.
260
Q

Differentials for Goodpastures

A
  • Wegener’s granulomatosis
  • SLE
  • Microscopic polyangiitis
  • Churg-Strauss syndrome
261
Q

Management for Goodpastures

A
  • Immunosuppresants
    • Corticosteroids
    • Cyclophosphamide
  • Plasmapheresis: to remove harmful antibodies
  • Dialysis may be required
  • Intubation and ventilation may be required
  • Bronchoscopy or angiography and embolisation: to control pulmonary haemorrhage
262
Q

Complications of Goodpastures

A
  • Chronic renal failure/ end-stage renal disease
  • Pulmonary haemorrhage
263
Q

Define Wegeners Granulomatosis (Granulomatosis with Polyangiitis)

A

A multi-system disorder of unknown causes characterised by necrotising granulomatous inflammation and vasculitis of small vessels.

It classically affects: the upper respiratory tract, the lower respiratory tracts and the kidneys.

264
Q

RF for Wegeners Granulomatosis

A
  • Genetic risk: variants in Proteinase 3 (PR3), SERPINA1 gene (affected in alpha-1-antitrypsin deficiency), and certain human leucocyte antigens (e.g. HLA-DP, HLA-DRB1-15) that are needed for antigen processing.
  • Inciting events:
    • Infections(e.g. staphylococcal infection)
    • Medications(e.g. use of hydralazine, minocycline, propylthiouracil or allopurinol among others): these have been shown to cause development of ANCA autoantibodies.
    • Alpha-1-antitrypsin (AAT): theoretical link with GPA because AAT is the natural inhibitor of PR3.
    • Environmental exposures: possible link with cigarette smoke and silica dust
265
Q

All Clinical manifestations of Wegeners Granulomatosis (systemic, Upper resp, Lungs, kidneys, other)

A

Systemic symptoms e.g.

  • Fever
  • Fatigue
  • Joint pain
  • Weight loss

Upper respiratory tract

  • Chronic pain caused by sinusitis
  • Bloody mucus due to ulcers within the nose
  • Nasal crusting
  • Saddle nose deformity: nose caves in
  • Hearing loss
  • Otitis media
  • Polychondritis (inflammation of cartilage)

Lungs

  • Breathing difficulty due to inflammation
  • Pleuritic pain
  • Wheeze
  • Ulcers can cause bloody coughing

Kidneys - rapidly progressing glomerulonephritis

  • Decreased urine production due to glomeruli death
  • Proteinuria
  • Haematuria
  • Increase in BP

Other

  • Vasculitic rash (palpable purpura): due to skin involvement
  • Hoarse voice: laryngeal involvement
  • Scleritis/ uveitis/ conjunctivitis: small vessels of eyes affected
  • Painful exophthalmos (bulging of eye) and diplopia: due to granuloma growing behind eyeball
  • Mononeuritis multiplex
  • Sensory neuropathy
  • Cranial nerve abnormalities
  • Gastrointestinal bleeding
  • Pericarditis or myocarditis
  • Arthralgia
266
Q

Investigations for Wegeners Granulomatosis

A
  • cANCA +ve
  • ESR and CRP raised
  • Raised WCC
  • Urinalysis - to look for proteinuria and haematuria
267
Q

Management for Wegeners Granulomatosis

A
  • Steroids combined with cyclophosphamide/ rituximab
  • Methotrexate/ azathioprine/ rituximab usually used for maintenance
  • Plasma exchange in patients with severe renal disease or pulmonary haemorrhage
268
Q

Prognosis for Wegeners Granulomatosis

A

Over 50% of patients with GPA should recover renal function and be dialysis independent. Renal involvement is associated with a poorer prognosis.

Approximately, 50% of patients have disease relapse with 5 years of diagnosis - commonly reoccurs due to the presence of cANCA.

269
Q

Define and overview of Pharyngitis/Tonsilitis

A

Acute pharyngitis is characterised by the rapid onset of sore throat and pharyngeal inflammation.

Acute tonsillitis refers to inflammation of the parenchyma of the palatine tonsils.

Both are most commonly due to a viral infection.

The literature does not clearly discriminate between acute tonsillitis and pharyngitis, often categorising both under ‘sore throat’.

270
Q

Epidemiology of Pahryngitis/tonsilitis

A

Acute tonsillitis is most common in children: children aged 5 to 10 are most often affected, with another peak between ages 15 and 20.

271
Q

RF for pharyngitis/tonsilitis

A

Young age, infected contact

272
Q

Pathophysiology/aetiology of pharyngitis/tonsilitis

A

Pharyngitis:

Infection of the pharynx or throat, which can develop if a virus e.g. adenovirus, rhinovirus, EBVetc move beyond the nose, and travel down into the pharynx.

Some bacteria e.g. group A streptococcus can also cause pharyngitis - “strep throat”.

Tonsillitis

If the infection spreads to involve the tonsils, it’s called tonsillitis.

Pathogens penetrate the tonsillar epithelium, causing local inflammation that results in oropharyngeal swelling, erythema, oedema and pain.

Causes of tonsillitis:

  • Viral tonsillitis: the most common; rhinovirus is the most common causative pathogen, followed by coronavirus and adenovirus
  • Bacterial tonsillitis: accounts for 10-30% of cases, with group A beta-haemolytic streptococci (GAS;strep. pyogenes) being the most common bacterial cause. Other bacterial causes include:
    • Haemophilus influenzae
    • Morazella catarrhalis
    • Staphylococcus aureus
  • Recurrent tonsillitis:staph. aureusis a common cause due to its antimicrobial resistance and persistence in the internal tissues of the tonsils
  • Non-infectious tonsillitis: rare and causes include GORD, chronic cigarette smoke, and hayfever
273
Q

Signs of pharyngitis/tonsilitis

A

Tonsillitis is characterised by features of pharyngitis, fever, malaise and lymphadenopathy.

  • Pyrexia: usually >38°C in tonsillitis
  • Red, inflamed and enlarged tonsils, with or without exudates (pus)
  • Anterior cervical lymphadenopathy
  • Evidence of dehydration if reduced oral intake: e.g. reduced skin turgor, dry mucous membranes
274
Q

Symptoms of pharyngitis/tonsilitis

A
  • Sore throat: usually sudden onset
  • Pain on swallowing
  • Loss of appetite
  • Fever
  • Malaise
  • Non-specific symptoms: headache, nausea, vomiting
275
Q

Investigations and diagnosis for pharyngitis/tonsilitis

A
  • Acute tonsillitis is primarily aclinical diagnosis
  • Investigations to consider:
    • Throat culture: gold-standard for definitively diagnosing bacterial tonsillitis; not routinely performed.
    • Rapid group A streptococcal (GAS) antigen test: a less sensitive alternative to throat culture
      • Only performed if the diagnosis of GASmust be confirmed with certainty,e.g. high risk of rheumatic fever, vulnerable people (very old and very young) or immunosuppressed
276
Q

Differentials for pharyngitis/tonsilitis

A

Infectious mononucleosis (glandular fever; due to EBV)
Epiglottitis

277
Q

Management for pharyngitis/tonsilitis

A
  • Most cases are self-limiting without antibiotics, even if bacterial, with symptoms lasting around 1 week.
  • Paracetamol and ibuprofen: as required, for pain and fever
  • Adequate intake of fluid: to avoid dehydration
  • Consider antibiotics: e.g. if high FeverPAIN (4 or 5) or Centor score (3 or 4)- antibiotics can be immediate or delayed
    • First line: phenoxymethylpenicillin
    • If penicillin allergy: clarithromycinorerythromycin
  • Arrange hospital admission:
    • Rarely requiredbut a few indications include inability to swallow, breathing difficulty, clinical dehydration, peri-tonsillar abscess, retropharyngeal abscess or sepsis
    • Corticosteroids: oral prednisolone or IV/IM dexamethasone may be appropriate if there are severe clinical features, e.g. significant oropharyngeal swelling, upper airway limitation, or inability to tolerate any oral intake
  • Tonsillectomy if:
    • Recurrent tonsillitis or complications (e.g. quinsy)
    • Obstructive sleep-disordered breathing in children <16 years
278
Q

Complications of pharyngitis/tonsilitis

A
  • Chronic tonsillitis
  • Acute otitis media
  • Peri-tonsillar abscess (quinsy): local abscess formation due to bacterial tonsillitis; associated with ‘hot potato’ voice, trismus (‘lockjaw’), and uvula displacement to the unaffected side
  • Parapharyngeal abscess
  • Scarlet fever
  • Acute rheumatic fever
  • Acute post-streptococcal glomerulonephritis
  • Post-streptococcal reactive arthritis
  • Post-tonsillectomy haemorrhage: considered a surgical emergency
279
Q

Define Acute Otitis Media

A

Acute otitis media (AOM) is defined by NICE as “the presence of inflammation in the middle ear, associated with an effusion and accompanied by the rapid onset of symptoms and signs of an ear infection”

It is a common complication of viral respiratory illnesses.

280
Q

Epidemiology of AOM

A
  • Most commonly affects children
  • Approximately 80% of all children will experience a case of otitis media during their lifetime
  • It is more commonly seen in winter and is often associated with an upper respiratory tract infection.
281
Q

RF for AOM

A
  • Children:6 to 24 months are most frequently affected
  • Attendance at daycare:increased exposure to pathogens
  • Bottle feeding:the absence of breastfeeding is a well-recognised risk factor
  • Family history
  • Craniofacial abnormalities
  • Gastroesophageal reflux disease
282
Q

Pathophysiology/aetiology of AOM

A

AOM most commonly affects children because viral infections are more common in this age group, and due to their shorter and more horizontal Eustachian tubes.

The bacteria enter from the back of the throat through the eustachian tube. Bacterial infection of the inner ear is often preceded by a viral upper respiratory tract infection.

AOM can be caused by both viral and bacterial pathogens, and often both are present at the same time.

Bacterial pathogens:

  • Streptococcus pneumoniae (most common)
  • Haemophilus influenzae
  • Moraxella catarrhalis
  • Staphylococcus aureus

Viral pathogens:

  • Respiratory syncytial virus
  • Rhinovirus
  • Adenovirus
  • Influenza
283
Q

Signs of AOM

A
  • Otoscopy findings:
    • A red or cloudy tympanic membrane
    • Bulging of the tympanic membrane
    • Middle ear effusion: air-fluid level behind the tympanic membrane
    • Tympanic membrane perforation may be present
  • Otorrhoea: discharge due to perforation of tympanic membrane
284
Q

Symptoms of AOM

A
  • Ear pain often associated with holding, tugging or rubbing of the ear in children
  • Reduced hearing
  • Recent upper respiratory tract infection
  • Balance issues and vertigo: if infection affects the vestibular system
  • Non-specific symptoms
    • Fever
    • Irritability and poor feeding
    • Vomiting
    • Sore throat
    • Cough and coryza
285
Q

Investigations for AOM

A
  • Otitis media is a clinical diagnosis
    • Examination: of ears and throat. Use otoscope to visualise the tympanic membrane - may appear bulging, red and inflamed. May be signs of discharge.
    • Triad of: bulging tympanic membrane, impaired mobility, and redness or cloudiness of the tympanic membrane
  • Imaging:
    • CT to confirm diagnosis
286
Q

Management of AOM

A
  • Observation: many cases are self-resolving
  • Analgesia: e.g. paracetamol or ibuprofen
  • Antibiotics: if not self-resolving, systemically unwell or suspected complications
    • First line:amoxicillin
    • Second line:co-amoxiclav if no improvement on amoxicillin
    • Penicillin allergy: macrolide, e.g. clarithromycin or erythromycin
    Antibiotics can be immediate or delayed (if symptoms don’t improve)
  • Certain patients may require admission:
    • Severe systemic infection
    • Suspected acute complications of AOM, such as mastoiditis or meningitis
    • ## Under the age of 3 months, with a temperature ≥38°C; also consider if 3-6 months and temperature ≥39°C
287
Q

Complications of AOM

A
  • Glue ear:persistent otitis media with effusion (fluid), also known as glue ear, often self resolves but may require grommet insertion
  • Tympanic membrane perforation:this is an indication for antibiotics and most cases self resolve after 2 months but occasionally may require myringoplasty
  • Mastoiditis:rare complication occurring when infection spreads into mastoid air cells. Requires IV antibiotics due to risk of intracranial infection
  • Meningitis or abscess: rare
  • Facial nerve palsy:rare and occurs due to inflammation causing compression of nerve VII
  • Chronic or recurrent infection
  • Hearing loss
288
Q

Define Sinusitis

A

Sinusitis is inflammation of the paranasal sinuses.

Acute sinusitis can last up to four weeks, subacute sinusitis lasts between 1 to 3 months, and chronic sinusitis lasts more than 3 months.

289
Q

Pathophysiology/aetiology of Sinusitis

A
  • Most cases of sinusitis are acute; and are the result of a viral infection. The invading pathogen often causes an inflammatory response. This causes the goblet cells to over secrete mucus which also leads to congestion. At the same time, immune cells try to fight off these pathogens, and it can create pus - a mixture of pathogens, immune cells, and dead tissue.
    • Viral pathogens: rhinovirus, parainfluenza virus, and influenza virus
    • Bacterial pathogens: streptococcus pneumoniae, haemophilus influenzae, moraxella catarrhalis, and staphylococcus aureus
  • Causes of subacute and chronic sinusitis include:
    • Acute sinusitis that doesn’t resolve quickly
    • Allergies e.g. dust, pollution, or even fungi like Aspergillus in some immunocompromised people.
  • Chronic hyperplastic sinusitis caused by:
    • Allergies leading to hyperplasia of the connective tissues of the sinuses. This can give rise to nasal polyps.
290
Q

Clinical manifestations of Sinusitis

A
  • Purulent rhinorrhoea
  • Facial pain
  • Headache
  • Fever
  • Voice changes
  • Change in smell and taste
  • Cough: due to mucus buildup
291
Q

Investigations for Sinusitis

A
  • Diagnosis is mainly based on clinical presentation
  • Investigations to consider:
    • Rhinoscopy: tube containing a camera is inserted into the nose; shows evidence of clogged up sinuses that may be filled with mucus or pus.
    • Sinus culture
    • CT/ X-ray/ MRI sinuses
292
Q

Management for Sinusitis

A
  • Sinusitis usually last 2-3 weeks and resolves without treatment
  • High dose steroid nasal spray: if no improvement
  • Antibiotics: if likely due to bacterial cause
    • First line: penicillin V (phenoxymethylpenicillin) for a 5 day course
    • Second line: co-amoxiclav
    • If penicillin allergy: clarithromycin, erythromycin (pregnancy), doxycycline
  • Sinus surgery: to allow drainage in cases of chronic or recurrent sinusitis
293
Q

Define Acute Epiglottitis

A

Epiglottitis refers to inflammation and localised oedema of the epiglottis, which can result in potentially life-threatening airway obstruction.

294
Q

Epidemiology of Acute Epiglottitis

A
  • Epiglottitis has a frequency of ~2 per 100,000 people per year
  • More commonly occurs in children
  • M>F
295
Q

RF for Acute Epiglottitis

A
  • Age: since the introduction of the HiB vaccination, its incidence has shifted from children to adults. The peak age of presentation is 6 to 12 years, although it can occur at any age.
  • Male gender
  • Unvaccinated
  • Immunocompromised
296
Q

Pathophysiology/aetiology of acute epiglottitis

A

Physiology:

Epiglottis prevents food from entering trachea, when eating. Other times, it remains open to allow air flow to trachea.

Pathology

Classically, epiglottitis is caused by a bacterial infection of the epiglottis byHaemophilus influenzae B(gram-negative coccobacillus) in children. Since the introduction of the HiB vaccination, its incidence has decreased.

Other organisms have become more common in the developed world, such asStreptococcus pneumoniaeandStreptococcus pyogenes.

Immune cells in the epiglottis tissue, detect the invading bacteria and release cytokines e.g. TNF-alpha.

The cytokines cause blood vessels to become more permeable to fluid which results in local inflammation or swelling. The epiglottis can swell to the point of completely obscuring the airway within hours of symptoms developing.

Epiglottitis can occur in adults but is rarely life-threatening as the adult airway is larger and has a better tolerance for supraglottic oedema.

297
Q

Signs of acute epiglottitis

A

Rapid onset within hours

  • Signs
    • Stridor
    • Muffled voice: voice box is located close to epiglottis
    • Respiratory distress: intercostal and subcostal recession (ribs show when breathing), tracheal tug, nasal flaring, accessory muscle use
    • Tripod position: a sign of respiratory distress
      • The patient leans forward and supports their upper body on their knees
    • Pyrexial: often a very high temperature ~40°C
    • Looks very unwell or ‘toxic’
298
Q

Symptoms of Acute Epiglottitis

A
  • Fever
  • Sore throat
  • Dysphagia
  • Dysphonia (stridor)
  • Drooling
  • Distress
299
Q

Investigations for acute epiglottitis

A

DO NOT examine the airway or distress the child as this could lead to catastrophic airway occlusion and respiratory arrest.

  • Primary investigations
    • Laryngoscopy:diagnostic and will demonstrate swelling and inflammation of the epiglottis or supraglottis. It is also therapeutic as intubation can be performed at the same time if needed
    • Lateral neck radiograph:securing the airway is the priority but, once done, an x-ray can be performed looking for thethumb sign (soft tissue shadow that looks like a thumb pressed into the trachea); also useful for excluding a foreign body.
300
Q

1st line management for Acute epiglottitis

A
  • Secure the airway:
    • Airway compromise:urgently contact anaesthetics for endotracheal intubation. If intubation fails due to excessive oedema, a surgical airway will be required, such as a cricothyrotomy
    • Airway maintained:if there is a low risk of obstruction, such as minimal respiratory distress, then intubation is not necessary and humidified oxygen is given
  • Nebulised adrenaline:may be used in an emergency to minimise laryngeal oedema prior to intubation
  • Intravenous antibiotics:typically a broad-spectrum antibiotic, such as ceftriaxone
301
Q

Complications for acute epiglottitis

A
  • Airway obstruction: occurs secondary to significant upper airway inflammation and oedema
  • Respiratory failure can lead to a respiratory acidosis
  • Mediastinitis:infection can track along the retropharyngeal space and involve the mediastinum, which is associated with a poor prognosis
  • Soft tissue involvement:cellulitis or abscess within the neck
302
Q

Define Croup

A

Croup, also known as laryngotracheobronchitis, is a viral upper respiratory tract infection most commonly by the parainfluenza virus.

303
Q

Epidemiology of Croup

A
  • Croup affects approximately 15% of children at some point, accounting for 5% of hospital admissions for children aged 6 months to 6 years of age
  • Presentation mainly in late autumn to winter
  • M>F
304
Q

Aetiology of Croup

A
  • Parainfluenza virus
  • Adenovirus
  • Influenza
  • Respiratory syncytial virus (RSV)
  • Bacterial croup e.g. laryngeal diptheria, exists but is significantly less common than viral croup.
305
Q

RF for Croup

A
  • Age: typical age of presentation is 6 months to 3 years, but can be seen up to 6 years of age
  • Male gender
  • Prematurity: particularly in children with a history of chronic lung disease of prematurity
  • Underlying respiratory disease
306
Q

Pathophysiology of Croup

A

The pathogen causes inflammation and swelling of the larynx, trachea, and large bronchi due to migration and infiltration of white blood cells.

It produces a characteristic barking cough, whilst inflammation, laryngeal oedema and secretions can lead to stridor and upper airway obstruction.

307
Q

Signs of Croup

A
  • Pyrexia
  • Stridor
  • Respiratory distress: intercostal and subcostal recession (ribs are visible on breathing), tracheal tug, nasal flaring, accessory muscle use
308
Q

Symptoms of Croup

A
  • Barking cough: worse at night
  • Difficulty in breathing
  • Hoarse voice
  • Fever
  • Coryza
  • Lethargy and agitation in severe disease
309
Q

Investigations for Croup

A

It is important not to distress the child as this could precipitate airway obstruction. This includes avoiding an ENT examination and invasive procedures, such as venepuncture.

  • Croup is aclinical diagnosisand does not usually require any further investigations.
  • The Westley score is a classification system used to assess the severity of croup.
  • Investigations to consider:
    • Neck X-ray:not routinely carried out in clinical practice, but the classical finding is thesteeple sign(tracheal narrowing).
310
Q

General mangement for croup

A

Most cases can be managed at home with simple supportive treatment (fluids and rest)

  • Admit patients, if
    • Moderate or severe croup
    • Haemodynamically significant congenital heart disease
    • < 3 months old
    • Inadequate fluid intake < 50-75%
311
Q

Complications of croup

A
  • Airway obstruction: upper airway inflammation and oedema can precipitate acute airway obstruction, which requires urgent nebulised adrenaline and consideration of intubation
  • Superinfection: croup superinfection can cause bacterial tracheitis, which is commonly associated with staphylococcus aureus, and is potentially life-threatening. It can also result in pneumonia